SUSIEG 2018 B Flashcards

1
Q

A physical therapist is treating a child with spastic cerebral palsy who is 3 years old cognitively but at a 6-month-old gross developmental level. What is an appropriate treatment activity for this child?
A. Reaching for a multicolored object while in an unsupported standing position
B. Reaching for a multicolored object while in an unsupported, guarded sitting position
C. Visually tracking a black and white object held 9 inches from his/her face
D. Reaching for a black and white object while in the supine position

A

Correct Answer: B
The appropriate task would include the 6-month-old gross developmental level activity of working on unsupported sitting. A multicolored object is appropriate for a 3-year-old cognitive level.

How well did you know this?
1
Not at all
2
3
4
5
Perfectly
2
Q

A patient is receiving mobilizations to regain normal mid thoracic extension. After three sessions, the patient complains of localized pain that persists for greater that 24 hours. What is the therapist’s best option?
A. Change mobilizations to gentle, low -amplitude oscillations to reduce the joint and soft tissue irritation
B. Continue with current mobilizations, followed by a cold pack to the thoracic spine
C. Place the physical therapy on hold and resume in 1 week
D. Change to self-stretching activities, because the patient does not tolerate mobilization

A

Correct Answer: A
Changing to low-amplitude oscillations will promote a decrease in the pain and tissue irritation. If pain persists for more than 24 hours, the soft tissue and joint irritation may progress.

How well did you know this?
1
Not at all
2
3
4
5
Perfectly
3
Q

A therapist wishes to use behavior modification techniques as part of a plan of care to help shape the behavioural responses of a patient recovering from traumatic brain injury (TBI).
What intervention is the BEST to use?
A. Use frequent reinforcements for all desired behaviors
B. Encourage the staff to tell the patient which behaviors are correct and which are not
C. Reprimand the patient every time an undesirable behavior occurs
D. Allow the patient enough time for self-correction of the behavior

A

Correct Answer: A
Behavioral modification is best achieved through use of positive reinforcements for all desired behaviors.

How well did you know this?
1
Not at all
2
3
4
5
Perfectly
4
Q

A patient with multiple sclerosis (MS) presents with dysmetria in both upper extremities.
Which of the following interventions is the BEST choice to deal with this problem?
A. 3-Ib weight cuffs to wrists during activities of daily living (ADL) training
B. Isokinetic training using low resistance and fast movement speeds
C. Pool exercises using water temperatures greater than 85°F
D. Proprioceptive neuromuscular facilitation (PNF) patterns using dynamic reversals with carefully graded resistance

A

Correct Answer: D
Dysmetria is a coordination problem in which the patient is unable to judge the distance or range of movement (overshoots or undershoots a target). Adding manual resistance with PNF can assist the patient in slowing down the movement and achieving better control.

How well did you know this?
1
Not at all
2
3
4
5
Perfectly
5
Q

Knee capsular tightness has limited a patient’s ability to attain full flexion. An INITIAL intervention a physical therapist can employ to restore joint motion should emphasize sustained mobilization in the loose-packed position. Which of the following is the BEST choice to use?
A. Anterior glide and external rotation of the tibia
B. Posterior glide and external rotation of the tibia
C. Posterior glide and internal rotation of the tibia
D. Anterior glide and internal rotation of the tibia

A

Correct Answer: C
Posterior glide and internal rotation are accessory motions necessary to increase knee flexion. Initial treatment should not result in pain, soreness, or diminished range of motion.

How well did you know this?
1
Not at all
2
3
4
5
Perfectly
6
Q

A patient recovering from cardiac transplantation for end-stage heart failure is referred for exercise training. The patient is receiving immunosuppressive drug therapy (cyclosporine and prednisone). What guidelines should the therapist follow when implementing an exercise program for this patient?
A. Require longer periods of warm-up and cool-down
B. Require short bouts of exercise
C. Eliminate all resistance training
D. Require a frequency of 2-3 times/week

A

Correct Answer: A
A patient recovering from cardiac transplantation will require longer periods of warm-up and cool-down because physiological responses to exercise and recovery take longer.

How well did you know this?
1
Not at all
2
3
4
5
Perfectly
7
Q

A patient presents with weakness and atrophy of the biceps brachin resulting from an open fracture of the humerus. The therapist reads a report of needle electromyography (EMG) of the biceps. What is the anticipated muscle response after the needle is inserted and prior to active contraction?
A. Polyphasic potentials
B. Interference patterns
C. Electrical silence
D. Fibrillation potentials

A

Correct Answer: C
Inserting an EMG needle into a normal muscle cause a burst of electrical activity (insertional activity) after which the muscle produces no sound (electrical silence).

How well did you know this?
1
Not at all
2
3
4
5
Perfectly
8
Q

A new staff physical therapist (PT) on the oncology unit of a large medical center receives a referral for strengthening and ambulation for a woman with ovarian cancer. She is undergoing radiation therapy after surgical hysterectomy. Her current platelet count is 17,000. What intervention is indicated for this patient at this time?
A. Active range-of-motion (AROM) exercises and activities of daily living (ADLs)
exercises
B. Aerobic exercise 3-5 days/week at 40-60%, one repetition maximum
C. Resistance training at 60%, one repetition maximum
D. Progressive stair climbing using a weighted waist belt

A

Correct Answer: A
AROM and ADL exercises are beneficial and safe for this patient.

How well did you know this?
1
Not at all
2
3
4
5
Perfectly
9
Q

A patient complains of increased pain and tingling in both hands after sitting at a desk for longer than 1 hour. The diagnosis is thoracic outlet syndrome (TOS). Which treatment would be the MOST effective physical therapy intervention?
A. Cardiovascular training using cycle ergometry to reduce symptoms of TOS
B. Stretching program for the pectoralis minor and scalenes
C. Strengthening program for the scalenes and sternocleidomastoids
D. Desensitization by maintaining the should in abduction, extension, and external rotation with the head turned toward the ipsilateral shoulder

A

Correct Answer: B
TOS is described as compression to the neurovascular structures in the scalene triangle, the area defined by the anterior and middle scalenes between the clavicle and the first rib. The compression is a result of a shortened pectoralis minor and scalene muscle. Therefore, a stretching program to these muscles to gain space in the scalene triangle is appropriate.

How well did you know this?
1
Not at all
2
3
4
5
Perfectly
10
Q

A patient with diabetes is exercising. The patient reports feeling weak, dizzy, and somewhat nauseous. The therapist notices that the patient is profusely and is unsteady when standing.
What is the therapist’s BEST immediate course of action?
A. Insist that the patient sit down until the orthostatic hypotension resolves
B. Have a nurse administer an insulin injection for developing hyperglycemia
C. Administer orange juice for developing hypoglycaemia
D. Call for emergency services; the patient is having an insulin reaction

A

Correct Answer: C
Hypoglycaemia, or abnormally low blood glucose, results from too much insulin (insulin reaction). It requires accurate assessment of symptoms and prompt intervention. Have the patient sit down and give an oral sugar (e.g., orange juice).

How well did you know this?
1
Not at all
2
3
4
5
Perfectly
11
Q

A patient with postpolio syndrome started attending a supervised outpatient exercise program. The patient failed to show up for follow-up sessions. The patient reported increased muscle pain and being too weak to get out of bed for the past 2 days. The patient is afraid to continue with the exercise class. What is the therapist’s BEST course of action regarding the patient’s exercise program?
A. Discharge the patient from the program because exercise is counterproductive in postpolio syndrome
B. Reschedule exercise workouts for early morning when there is less fatigue
C. Decrease the intensity and duration, but maintain a frequency of 3 time/week
D. Decrease the frequency to once a week for an hour session, keeping the intensity moderate

A

Correct Answer: C
Clinical manifestations of postpolio syndrome include myalgias, new weakness as well as atrophy and excessive fatigue with minimal activity, Nonexhaustive exercise and general body conditioning are indicated. A change in the exercise prescription (intensity and duration) is warranted.

How well did you know this?
1
Not at all
2
3
4
5
Perfectly
12
Q

A therapist has elected to use continuous inductive coil short wave diathermy (SWD) as one of the interventions in managing hip pain. Use of other thermal or electrical modalities were either ineffective or contraindicated. Which patient would be a candidate for use of short wave diathermy?
A. A morbidly obese patient
B. A patient with Type 1 diabetes who uses an insulin pump
C. An 11 year-old boy with a slipped capital femoral epiphysis
D. A patient with ankylosing spondylitis on high doses of NSAIDS and DMARDS

A

Correct Answer: D
Ankylosing spondylitis is a form of arthritis that primarily affects the spine and other joints including the hip. Pain, stiffness, and inflammation can result. A physical therapy treatment plan can incorporate deep heating for muscle relaxation followed by stretching, posture management, and other exercises. Inductive SW D is preferred here as it penetrates more deeply than capacitive SWD. The use of the medications indicated will have no bearing on the application of SWD.

How well did you know this?
1
Not at all
2
3
4
5
Perfectly
13
Q

A patient is recovering from open heart surgery (sternotomy and coronary artery bypass).
The PT is supervising the patient’s outpatient exercise program at 8 weeks postsurgery.
What guideline should be followed regarding the use of moderate to heavy weights during resistance training?
A. Should include upper body exercises only
B. Is contraindicated during the first two months
C. Should be based on 60%-80%, one repetition maximum initially
D. Can be included if resistance training is once a week

A

Correct Answer: B
Resistive training after cardiothoracic surgery is restricted to 5 to 8 pounds for the first 5 to 8 weeks. Moderate to heavy resistance exercises are contraindicated.

How well did you know this?
1
Not at all
2
3
4
5
Perfectly
14
Q

14.A patient with asthma is taking a drug from the sympathomimetic group, albuterol (Ventolin). What is the MOST important effect of this medication?
A. Increases airway resistance and decreases secretion production
B. Reduces airway resistance by reducing bronchospasm
C. Increases heart rate (HR) and BP to enhance training effect during aerobic activity
D. Reduces bronchial constriction and high blood pressure (BP) that accompanies exercise

A

Correct Answer: B
Sympathomimetics are a class of drugs that mimics the effects of stimulation of body organs and structures by the sympathetic nervous system. Albuterol (Ventolin) has the primary action of reducing airway resistance by a decrease in bronchospasm.

How well did you know this?
1
Not at all
2
3
4
5
Perfectly
15
Q

A PT decides to exercise a patient with lower extremity lymphedema using aquatic therapy.
Hydrostatic pressure exerted by the water can be expected to do which of the following?
A. Increase cardiovascular demands at rest and with exercise
B. Reduce effusion and assist venous return
C. Provide joint unloading and enhance ease of active movement
D. Increase resistance as speed of movement increases

A

Correct Answer: B
The pressure exerted by water on an immersed object is equal on all surfaces (pascal’s law). As the depth of immersion increases, so does hydrostatic pressure. Increased pressure limits effusion, assists venous return, and can induce bradycardia.

How well did you know this?
1
Not at all
2
3
4
5
Perfectly
16
Q

16.A therapist determines that a patient is walking with a backward trunk lean with full weight on the right leg. The patient also demonstrates great difficulty going up ramps. What is the BEST intervention to remediate this problem?
A. Strengthen hip extensors through bridging
B. Stretch hip abductors through side-lying positioning
C. Strengthen knee extensors with weights, using 80%, one repetition maximum
D. Stretch hip flexors through prone-lying positioning

A

Correct Answer: A
Backward trunk lean (gluteus maximus gait) is the result of a weak gluteus maximus, It causes increased difficulty going up stairs or ramps. Functional-training exercises such as bridging are indicated.

How well did you know this?
1
Not at all
2
3
4
5
Perfectly
17
Q

17.A patient has developed a thick eschar secondary to a full-thickness burn. What is the antibacterial agent MOST effective for infection control for this type of burn?
A. Sulfamylon
B. Nitrofurazone
C. Panafil
D. Silver nitrate

A

Correct Answer: A
Sulfamylon penetrates through eschar and provides antibacterial control.

How well did you know this?
1
Not at all
2
3
4
5
Perfectly
18
Q

18.An elderly person has lost significant functional vision over the past 4 years and complains of blurred vision and difficulty reading. The patient frequently mistakes images directly in ront of her, especially in bright light. When walking across a room, the patient is able to locate items in the environment using peripheral vision when items are located to both sides.
Based on these findings, what is the visual condition this patient is MOST likely experiencing?
A. Glaucoma
B. Cataracts
C. Homonymous hemianopsia
D. Bitemporal hemianopsia

A

Correct Answer: B
Cataracts, which cause a clouding of the lens, result in a gradual loss of vision; central vision is lost first, then peripheral.

How well did you know this?
1
Not at all
2
3
4
5
Perfectly
19
Q

19.A patient presents with complaints of pain and difficulty with ADL that is consistent with carpal tunnel syndrome. What is the BEST test to identify the cause of symptoms in this patient?
A. Pronator teres syndrome test
B. Ulnar nerve tension test
C. Allen’s test
D. Phalen’s test

A

Correct Answer: D
Phalen’s test places stress on the compartment where the median nerve passes into the hand, so this test is typically positive for patients with carpal tunnel syndrome.

How well did you know this?
1
Not at all
2
3
4
5
Perfectly
20
Q

20.A patient incurred a right CVA 1 month ago and demonstrates moderate spasticity in the left upper extremity (predominantly increased flexor tone), The major problem at this time is a lack of voluntary movement control. There is minimal active movement, with ¼ inch subluxation of the shoulder. What initial treatment activity is the BEST choice for this patient?
A. Sitting, left active shoulder protraction with extended elbow and shoulder flexed to 90°
B. Sitting, weight bearing on extended left upper extremity, weight shifting
C. Quadruped, rocking from side to side
D. PNF D2 flexion pattern, left upper extremity

A

Correct Answer: B
Sitting, weight bearing, and rocking on an extended left upper extremity will help to decrease the flexor tone. It also provides joint compression (approximation) at the shoulder, which will help maintain shoulder position and stimulate stabilizing muscles.

How well did you know this?
1
Not at all
2
3
4
5
Perfectly
21
Q

21.An older patient complains of pain in the right hip region. The therapist suspects hip osteoarthritis based on the patient’s subjective symptoms. What clinical test is the BEST choice to confirm this diagnosis?
A. Scouring test
B. Thomas test
C. Craig’s test
D. Posterior impingement test

A

Correct Answer: A
A positive scouring test would be a consistent finding for a patient who has osteoarthrosis of the hip joint. It compresses the joint.

How well did you know this?
1
Not at all
2
3
4
5
Perfectly
22
Q

22.The therapist in the photograph is testing which muscle?
A. Upper trapezius
B. Middle deltoid
C. Supraspinatus
D. Anterior deltoid

A

Correct Answer: C
The muscle being tested is the supraspinatus. The empty-can position puts the supraspinatus muscle in its most effective position for contraction. Weakness may be a result of inflammation, neuropathy of the suprascapular nerve, or a tendon tear.

How well did you know this?
1
Not at all
2
3
4
5
Perfectly
23
Q

23.During an examination, a patient demonstrates large-amplitude, sudden flailing motions of the arm and leg on one side of the body with primary involvement of axial and proximal joint muscles. What clinical term BEST describes the patient’s behaviors?
A. Chorea
B. Intention tremor
C. Hemiballismus |
D. Athetosis

A

Correct Answer: C
Hemiballismus refers to sudden, jerky, forceful, and flailing involuntary movements on one side of the body (“Hemi” was a clue).

How well did you know this?
1
Not at all
2
3
4
5
Perfectly
24
Q

24.A patient is referred to physical therapy after an antero- inferior dislocation of the right shoulder. What positive examination finding is expected as a result of this dislocation?
A. Weak rhomboids
B. Positive drop arm test
C. Positive Neer’s test
D. Weak deltoids

A

Correct Answer: D
Because of the anatomical position of the axillary nerve, it can be damaged by an antero-inferior dislocation at the glenohumeral joint. This results in weak deltoids.

How well did you know this?
1
Not at all
2
3
4
5
Perfectly
25
Q

25.A team of researchers investigates the use of constraint induced movement therapy on patients with chronic stroke (>1 year poststroke) using a multicentre randomized controlled trial (RCT). What are the specific characteristics of this type of research design?
A. A sample of convenience for the intervention group
B. Alternating experimental and control conditions for a subject
C. Random assignment to an experimental or control group
D. Random assignment to matched cohort groups

A

Correct Answer: C
An RCT uses a randomization process to assign subjects to either an experimental group(s) or a control (comparison) group. Subjects in the experimental group receive the intervention and are then compared with subjects in the control group who do not receive the intervention. A large multicentre RCT study with large numbers of patients provides the highest level of scientific rigor and evidence.

How well did you know this?
1
Not at all
2
3
4
5
Perfectly
26
Q

26.A therapist has been asked to give an in-service presentation to staff aides on safe guarding techniques in a nursing home. The patients are at risk for falls. How should the therapist BEST prepare for this talk?
A. Provide a questionnaire to a random sampling of participants 1 week before the scheduled presentation
B. Provide a questionnaire to all participants 2 weeks before the scheduled session
C. Survey the audience a day before the scheduled session
D. Survey the audience at the scheduled session

A

Correct Answer: B
A questionnaire to all participants represents the best method of needs assessment in this situation.

How well did you know this?
1
Not at all
2
3
4
5
Perfectly
27
Q

27.A therapist is treating a patient with left hemiplegia and profound visuospatial perceptual deficits. What is the BEST strategy to use initially to assist this patient in the relearning of motor tasks?
A. Simplify and restructure the environment and minimize distractions
B. Maximize use of demonstration and gesture
C. Minimize use of verbal cues
D. Encourage independent practice

A

Correct Answer: A
Working in a closed environment in which clutter and distractions are minimized or eliminated is the best choice for this patient during initial training.

How well did you know this?
1
Not at all
2
3
4
5
Perfectly
28
Q

28.A 2-week-old infant born at 27 weeks gestation with hyaline membrane disease is referred for a physical therapy consult. Nursing reports that the child *desaturates to 84% with handling” and has minimal secretions at present. What is the therapist’s BEST course of action?
A. Provide suggestions to nursing for positioning for optimal motor development
B. Put the PT consult on hold because the child is too ill to tolerate exercise
C. Delegate to a physical therapy assistant (PTA) a maintenance program of manual techniques for secretion clearance.
D. Perform manual techniques for secretion clearance, 2-4 hours daily, to maintain airway patency.

A

Correct Answer: A
Excessive handling of a premature infant can cause oxygen desaturation. It is in the best Cinterests of the infant to limit the number of handlers. The PT’s role should be to assist nursing in developing positioning schedules, positions for feeding, infant stimulation activities, etc.

How well did you know this?
1
Not at all
2
3
4
5
Perfectly
29
Q

29.A patient sustained a valgus stress to the left knee while skiing. The orthopedist found a positive McMurray’s test and a positive Lachman’s stress test. The patient has been referred to physical therapy for conservative management of this problem. What is the BEST intervention for the subacute phase of rehabilitation?
A. Open-chain exercises of the hip extensors and hamstrings to inhibit anterior translation of the femur on the tibia.
B. Closed-chain functional strengthening of the quadriceps femoris and hamstrings, emphasizing regaining terminal knee extension.
C. Closed-chain functional strengthening of the quadriceps femoris and hip abductors to promote regaining terminal knee extension.
D. Open-chain strengthening of the quadriceps femoris and hip adductors to inhibit anterior translation of the tibia on the femur.

A

Correct Answer: B
The evaluation is suggestive of an unhappy triad injury. Closed-chain exercises are emphasized during the subacute phase to enhance functional control of the muscles surrounding the knee. Terminal extension must be achieved during this stage if normal function is to occur.

How well did you know this?
1
Not at all
2
3
4
5
Perfectly
30
Q

30.A patient with pain in the left lateral face and head is found to have limited active and passive mouth opening range of motion. However, passive lateral deviation is full to both sides. What is the likely reason for the limitation in mouth opening range of motion?
A. An anteriorly displaced disc with reduction in the left temporomandibular joint
B. Decreased flexibility in the muscles of mastication on the left
C. Capsular restriction of the left temporomandibular joint
D. An anteriorly displaced disc without reduction in the left temporomandibular joint

A

Correct Answer: B
Mouth opening requires lengthening of the muscles of mastication as the body of the mandible moves away from the upper palate. Lateral deviation does not require a significant lengthening in the muscles of mastication, as the primary motion of the mandible is a slight anterior translation of the mandibular condyle without increasing the distance between the body of the mandible and the upper palate.

How well did you know this?
1
Not at all
2
3
4
5
Perfectly
31
Q

31.A patient with TBI has a convulsive seizure during a therapy session. The patient has lost consciousness and presents with tonic-clonic convulsions of all extremities. What is the therapist’s BEST response?
A. Position in supine-lying with head supported with a pillow, and wait out the seizure
B. Wrap the limbs with a sheet to prevent self-harm, position in supine-lying and call for emergency assistance.
C. Position in side-lying, check for an open airway, and immediately call for emergency
assistance
D. Initiate rescue breathing immediately and call for help to restrain the patient

A

Correct Answer: C
This is an emergency situation. In order to prevent aspiration, turn the head to the side or position in side-lying. Check to see whether the airway is open, and then call for emergency assistance. Wait for tonic-clonic activity to subside before initiating artificial ventilation, if needed.

How well did you know this?
1
Not at all
2
3
4
5
Perfectly
32
Q

32.A therapist wants to know whether neurodevelopmental treatment (NDT) handling techniques produce an improvement in independent rolling that lasts longer than 30 minutes.
In this study, rolling is what type of variable?
A. Control variable
B. Independent variable
C. Dependent variable
D. Intervening variable

A

Correct Answer: C
The dependent variable is the change or difference in behavior (in this example, rolling) that results from the intervention.

How well did you know this?
1
Not at all
2
3
4
5
Perfectly
33
Q

33.An adult with no significant past medical history presents to the emergency room with complaints of fever, shaking chills, and a worsening productive cough. The patient has chest pains over the posterior base of the left thorax, which are made worse on inspiration. What would be an expected physical finding for this patient?
A. Symmetrical breathing
B. Crackles over the left thorax
C. Increased chest excursion
D. Slowed respiratory rate

A

Correct Answer: B
Crackles are a typical finding over the area correlating with an infiltrate.

How well did you know this?
1
Not at all
2
3
4
5
Perfectly
34
Q

34.A patient is referred to physical therapy for functional gait difficulties. The patient is unable to take a normal step and drags the left foot. Examination reveals muscle weakness with fasciculations in the left lower leg. For what should the therapist examine?
A. Decreased tone and hyporeflexia
B. Muscle spasms and positive Babinski
C. Increased tone and hyperreflexia
D. Dyssynergia and timing deficits

A

Correct Answer: A
Muscle weakness with fasciculations is symptomatic of a lower motor neuron (LMN) lesion. Other signs and symptoms of a LMN lesion include hypotonia or flaccidity, hyporeflexia, or absent reflexes and neurogenic atrophy.

How well did you know this?
1
Not at all
2
3
4
5
Perfectly
35
Q

35.An elderly patient with hyperthyroidism is referred to physical therapy following a period of prolonged bedrest. What should the therapist be alert for when monitoring exercise of this patient?
A. Decreased heart rate and blood pressure
B. Tachycardia and dyspnea
C. Muscle weakness and joint pain
D. Arrhythmias and bradycardia

A

Correct Answer: B
Hyperthyroidism is a hypermetabolic state and is associated with exercise intolerance and impaired cardiopulmonary function, Symptoms include dyspnea, fatigue, tachycardia, and arrhythmia. In older people there is increased risk of aggravating pre-existing heart disease (e.g., atrial fibrillation, angina, and myocardial infarction).

How well did you know this?
1
Not at all
2
3
4
5
Perfectly
36
Q

36.A patient strained the lower back muscles 3 weeks ago, and now complains of pain (6/10).
Upon examination, the therapist identifies bilateral muscle spasm from T10-L4. The therapist elects to apply interferential current to help reduce pain and spasm. What is the BEST electrode configuration in this case?
A. Four electrodes, with current flow perpendicular to the spinal column
B. Two electrodes, with current flow perpendicular to the spinal column
C. Four electrodes, with current flow diagonal to the spinal column
D. Two electrodes, with current flow parallel to the spinal column

A

Correct Answer: C
The crisscrossed electrode configuration allows: (1) a greater area to be treated and (2) current interference to occur between the frequencies of the two circuits because of the diagonal pattern.

How well did you know this?
1
Not at all
2
3
4
5
Perfectly
37
Q

37.An infant is independent in sitting, including all protective extension reactions, and can pull-to-stand through kneeling, cruise sideways, and stand alone. The infant still demonstrates plantar grasp in standing. What is this infant’s approximate chronological age?
A. 6 months
B. 5 months
C. 8-9 months
D. 10-15 months

A

Correct Answer: C
The 8 to 9-month-old will be able to pull-to-stand, stand alone, and cruise sideways, but because he/she is not yet walking, may still exhibit plantar grasp in the standing position,

How well did you know this?
1
Not at all
2
3
4
5
Perfectly
38
Q

38.A retired bus driver has experienced increasing frequency of low back pain over the past 10 years. The patient states that nonsteroidal anti-inflammatory drugs (NSAIDs) help to relive the symptoms, but there is always a nagging-type pain, the patient reports significant stiffness in the morning that dissipates by noon after exercising and walking, Pain is exacerbated with frequent lifting and bending activities, as well as sitting for long period.
What should the physical therapy plan of care emphasize?
A. Modalities to reduce pain, postural re-education, and dynamic stabilization exercises.
B. Postural re-education, soft tissue mobilization, and dynamic stabilization.
C. Modalities to reduce pain, joint mobilization, and lumbar extension exercises.
D. Joint mobilization, soft tissue mobilization, and flexion exercises.

A

Correct Answer: B
This is a long-term degenerative and postural dysfunction that is manageable with medication and proper physical activity. Therefore, the most effective use of treatment time should emphasize regaining normal postural alignment and functional ADLs.

How well did you know this?
1
Not at all
2
3
4
5
Perfectly
39
Q

39.A patient presents to physical therapy with a primary complaint of low back pain and right lower extremity radicular symptoms extending distally to the calf of 2 weeks duration.
Current pain intensity is rated as 2/10 with rest and 5/10 during lumbar extension movements. What is the strongest prognostic indicator that would affect the clinical outcome?
A. 2/10 pain intensity with rest
B. Current symptom duration (2 weeks)
C. Lower extremity radicular symptoms
D. 5/10 pain intensity during lumbar extension

A

Correct Answer: C
The presence of lower extremity radicular symptoms in patients experiencing low back pain is a strong negative prognostic indicator for achieving good clinical outcomes.

How well did you know this?
1
Not at all
2
3
4
5
Perfectly
40
Q
  1. An elderly patient with diabetes and bilateral lower extremity amputation is to be discharged from an acute care hospital 2 weeks postsurgery. The incisions on the residual limbs are not healed and continue to drain. The patient is unable to transfer because the venous graft sites in the upper extremities are painful and not fully healed. Endurance out-of-bed is limited.
    What is the BEST choice of discharge destination for this patient?
    A. Skilled nursing facility
    B. Custodial care facility
    C. Home
    D. Rehabilitation hospital
A

Correct Answer: A
A skilled nursing facility is the best facility because the patient continues to require nursing care for the open wounds. Initiation of physical therapy when this patient is able is also available

How well did you know this?
1
Not at all
2
3
4
5
Perfectly
41
Q

41.A patient currently being seen for low back pain awoke one morning with drooping left facial muscles and excessive drooling. The patient was recovering from a cold and had experienced an earache in the left ear during the previous 2 days. The therapist suspects Bell’s palsy. What cranial nerve test can confirm this diagnosis?
A. Taste over the posterior tongue, and having the patient protrude the tongue
B. Taste over the anterior tongue, and having the patient raise the eyebrows and puff the cheeks
C. Corneal reflex and stretch reflexes of facial muscles
D. Trigger points for pain, especially over the temporomandibular joint (TMJ).

A

Correct Answer: B
Bell’s palsy is a lower motor neuron lesion affecting the branches off the facial nerve, CN VII. Examination of the motor function of the muscles of facial expression (i.e., raise eyebrows, show teeth, smile, close eyes tightly, puff cheeks) and taste over the anterior tongue will reveal deficits of CN VII function.

How well did you know this?
1
Not at all
2
3
4
5
Perfectly
42
Q

42.A PTA is assigned to ambulate a patient with a 10-year history of Parkinson’s disease (PD).
What should the PT instruct the PTA to watch for?
A. Wider steps and increased double support time
B. An abnormally wide base of support
C. Decreased trunk rotation with shorter steps
D. Unsteady, uneven gait with veering to one side

A

Correct Answer: C
Gait changes characteristic of PD include loss of arm swing and reciprocal trunk movement, shuffling gait with shorter steps, and festinating gait (an abnormal and involuntary increase in the speed of walking).

How well did you know this?
1
Not at all
2
3
4
5
Perfectly
43
Q

43.A patient with congestive heart failure (CHF) is on a regimen of diuretics (chlorotiazide).
What are the potential adverse effects of this medication that the PT should be alert for?
A. Hyperkalemia and premature ventricular contractions (PVCs).
B. Myalgia and joint pains
C. Orthostatic hypotension and dizziness
D. Reflex tachycardia and unstable BP

A

Correct Answer: C
Thiazide diuretics are used to manage mild to moderate hypertension. Adverse side effects include orthostatic hypotension and dizziness, along with drowsiness, lethargy and weakness.
These represent a safety risk during functional training and gait.

How well did you know this?
1
Not at all
2
3
4
5
Perfectly
44
Q

44.A patient with a traumatic injury to the right hand had a flexor tendon repair to the fingers
When should physical therapy intervention being following this type of repair?
A. After the splint is removed in 4-6 weeks to allow ample healing time for the repaired tendon
B. After the splint is removed in 2-3 weeks to allow full AROM of all affected joints
C. Within a few days after surgery to allow for early initiation of strengthening exercises
D. Within a few days after surgery to preserve tendon gliding

A

Correct Answer: D
Early passive and active assistive exercises promote collagen remodelling to allow free tendon gliding

How well did you know this?
1
Not at all
2
3
4
5
Perfectly
45
Q

45.A female patient complains of intermittent pain in the right sacroiliac region. There was an insidious onset approximately 4 months ago, Pain has gradually worsened and is now fairly constant and does not vary much with activity or movement. Active motion assessment of the lumbar spine reveals no change in symptoms with movement. Sacroiliac provocation tests are negative. The patient is mildly tender over the right sacroiliac region. What is the MOST likely diagnosis for this patient?
A. Sacroiliac joint sprain
B. Multifidus muscle strain
C. Ovarian cyst
D. Right L5/S1 facet joint arthrosis

A

Correct Answer: C
An ovarian cyst can refer pain to the sacroiliac region and is more likely to cause constant pain that dies not vary much with activity. Symptoms are not likely to be reproduced with a musculoskeletal examination.

How well did you know this?
1
Not at all
2
3
4
5
Perfectly
46
Q
  1. The therapist suspects that a patient recovering from a middle cerebral artery stroke is exhibiting a pure hemianopsia. What test should be used to confirm the hemianopsia?
    A. Penlight held approximately 12 inches from the eyes and moved to the extremes of gaze right and left
    B. Penlight held 6 inches from the eyes and moved inward toward the face
    C. Visual confrontation test with a moving finger
    D. Distance acuity chart placed on a well-lighted wall at patient’s eye level 20 feet away
A

Correct Answer: C
Visual field is examined using the confrontation test. The patient sits opposite the therapist and is instructed to maintain his/her gaze on the therapist’s nose. The therapist slowly brings a target (moving finger or pen) in the patient’s field of view alternately from the right or left sides. The patient indicates when and where he/she first sees the target.

How well did you know this?
1
Not at all
2
3
4
5
Perfectly
47
Q

47.An overweight adult patient complains of right anterior hip and knee pain while walking, especially when weight bearing on the right. Lumbar AROM is normal and pain free. Right hip AROM and PROM are limited compared to the left. Right knee AROM and PROM are full and pain free. There is no pain with resisted testing at the right hip or right knee. The scouring test reproduces the patient’s hip and knee symptoms. Hip joint distraction relieves these symptoms. Based on the above findings, what is the MOST likely diagnosis?
A. Trochanteric bursitis
B. Patellofemoral syndrome
C. Piriformis strain
D. Hip degenerative joint disease.

A

Correct Answer: D
Scouring test or compression at the hip joint reproduces the patient’s symptoms, suggesting a joint problem at the hip. The scouring test can also result in referred pain to the knee. Weight bearing that compressed the hip joint also reproduces the patient’s symptoms. Distraction of the hip joint surfaces relieves symptoms, suggesting hip DJD. The anterior knee region is a common pain referral site for the hip joint. In addition, the overweight condition supports the diagnosis of hip DJD due to excessive compressive forces over an extended period of time.

How well did you know this?
1
Not at all
2
3
4
5
Perfectly
48
Q

48.A therapist investigated the accuracy of pulse oximetry estimates during exercise.
Correlational analysis measured the strength of the relationship between two types of ear probe-equipped pulse oximeters during heavy cycle exercise under hypoxic conditions. The investigator found measured arterial oxyhemoglobin saturation (%H602) level to have a correlation of 0.89 at high saturation but only 0.68 at low saturation levels. How should the therapist interpret these results?
A. During heavy exercise, oxygen saturation levels should be interpreted cautiously
B. Both devices are highly accurate at all saturation levels
C. Accuracy of the measurements increases at higher saturation levels
D. Both devices are only moderately accurate

A

Correct Answer: C
The result of the study indicates that the correlation between the two types of oximiters was high when oxygen saturation levels were high (0.89), but only moderate (0.68) at low oxygen saturation levels.

How well did you know this?
1
Not at all
2
3
4
5
Perfectly
49
Q

49.A college soccer player sustained a hyperextension knee injury when kicking the ball. The patient was taken to the emergency room of a local hospital and was diagnosed with “knee sprain.” The player was sent to physical therapy the next day for rehabilitation. As part of the examination to determine the type of treatment plan to implement, the therapist conducted the test shown in the figure. Based on the test picture, the therapist is examining the integrity of which structure?
A. Iliotibial band
B. Posterior cruciate ligament
C. Anterior cruciate ligament
D. Medial meniscus

A

Correct Answer: C
The test shown in the figure is Lachman’s stress test to determine the integrity of the anterior cruciate ligament

How well did you know this?
1
Not at all
2
3
4
5
Perfectly
50
Q

50.A patient is seen in physical therapy 2 days after a motor vehicle accident. The chief complaints are headaches, dizziness, neck pain with guarding, and a “sensation of a lump in the throat.” Plain film x-rays were read as negative. The therapist should refer this patient for what type of imaging?
A. Second series of plain film x-rays
B. T2 magnetic resonance imaging (MRI)
C. Computed tomography (CT) scan
D. Myelogram

A

Correct Answer: C
The primary concern of the therapist is to rule out strong suspicions of an upper cervical spine fracture, CT scan is still preferred for assessing cortical bone, especially spinal fractures.

How well did you know this?
1
Not at all
2
3
4
5
Perfectly
51
Q

51.During a test of upper extremity rapid alternating movement (RAM), the movements of the hands and elbows become irregular with wider excursions than expected, As speed is increased, the movements become more disorganized. These findings are indicative of:
A. Brainstem dysfunction
B. Lower motor neuron weakness
C. Cerebellar dysfunction
D. Upper motor neuron weakness

A

Correct Answer: C
Cerebellar dysfunction is characterized by classic cerebellar movement disturbances of dyssynergia (in this case), dysmetria and dysdiadochokinesia. Movement decomposition is velocity dependent, with greater disturbances in movement control at higher speeds.

How well did you know this?
1
Not at all
2
3
4
5
Perfectly
52
Q

52.A patient presents with a persistently downwardly rotated and adducted scapula during humeral elevation. The plan of care includes stretching and strengthening to improve range of motion. What muscles should be stretched and strengthened?
A. Stretching rhomboid muscles and strengthening serratus anterior muscle
B. Stretching pectoralis major and strengthening rhomboid muscles
C. Stretching pectoralis minor and strengthening trapezius muscles
D. Stretching serratus anterior and strengthening levator scapula and lower trapezius

A

Correct Answer: A
The rhomboids are scapular downward rotators, scapular adductors, and scapular elevators.
Insufficient length of the rhomboids would limit upward scapular rotation, and stretching to restore proper length of these muscles would help to promote upward scapular rotation.

How well did you know this?
1
Not at all
2
3
4
5
Perfectly
53
Q

53.A patient is recovering from stroke and, at 4 months, is ambulating with a straight cane for household distances. During outpatient physical therapy, the therapist has the patient practice walking with no assistive device. Recurvatum is observed that worsens with continued walking. What is the therapist’s BEST choice for intervention?
A. Give the patient a small-based quad cane (SBQC) to improve stability and have him/her practice AROM in supine
B. Exercise the quadriceps using isokinetic resistance at higher loads and increasing speeds
C. Practice isolated small-range quadriceps eccentric control work in standing and continue with the straight cane
D. Give the patient a KAFO to control the hypertension and a hemi-walker.

A

Correct Answer: C
Eccentric quadriceps control work (closed-chain exercises) is indicated to reduce recurvatum.
The patient should continue with a straight cane until able to walk without the device and
recurvatum.

How well did you know this?
1
Not at all
2
3
4
5
Perfectly
54
Q

54.A patient recovering from an incomplete spinal cord injury at the L3 level (ASIA scale D) ambulates with bilateral Lofstrand crutches. The patient reports great difficulty going down ramps with unsteady; wobbly knees. What is the BEST intervention to use with this patient?
A. Prolonged icing to reduce hamstring pain
B. Stretching using a posterior resting splint for tight plantar flexors
C. Progressive resistance training for the quadriceps
D. Biofeedback training to reduce knee extensor spasticity

A

Correct Answer: C
A spinal cord injury at the level of L3 affects knee extensors. ASIA scale D means the injury is incomplete, with at least half of the key muscles below the neurological level having a muscle grade of 3 or more, A weak knee will wobble or buckle going down stairs or ramps. It is the result fo weak quadriceps or knee flexor contracture. Strengthening exercises using progressive resistance training for the quadriceps are indicated.

How well did you know this?
1
Not at all
2
3
4
5
Perfectly
55
Q

55.Under HIPAA rules, to whom is it illegal to release protected health information (PHI) without a competent patient’s consent?
A. A State Agency responsible for investigating suspected abuse
B. The insurance company that is paying for the patient’s treatment
C. Another health care provider involved in the care of the patient
D. The patient’s spouse

A

Correct Answer: D
A spouse does not have the legal right to the patient’s information without the patient’s consent

How well did you know this?
1
Not at all
2
3
4
5
Perfectly
56
Q
  1. With respect to a worker’s sitting posture, the greatest reduction in lumbar spine compression forces would be achieved by:
    A. Eliminating armrests on the chair
    B. Decreasing the chair backrest-seat angle to 85°
    C. Increasing the chair backrest-seat angle to between 90° and 110°
    D. Using a 2-inch gel seat cushion
A

Correct Answer: C
Maximal reduction of lumbar disc pressure can be achieved by increasing the angle between the seat pan and the chair backrest to between 90° and 110°, using armrests for support, or adding a lumbar support. Combining the effects of all three provides the best solution.

How well did you know this?
1
Not at all
2
3
4
5
Perfectly
57
Q

57.A baseball pitcher is referred to physical therapy with progressive posterior shoulder pain and weakness of the shoulder abductors and lateral rotators. The therapist notices muscle wasting superior and inferior to the scapular spine. The patient’s problem is MOST LIKELY attributable to damage involving the:
A. Spinal accessory nerve
B. Scalene muscles
C. Suprascapular nerve
D. Long head of the biceps brachii

A

Correct Answer: C
Microtrauma to the suprascapular nerve can occur with repetitive activities involving shoulder
“cocking” and follow-through resulting in inflammation and muscle weakness of the muscles supplied by the suprascapular nerve (the supraspinatus and infraspinatus).

How well did you know this?
1
Not at all
2
3
4
5
Perfectly
58
Q

58.An elderly patient presents with severe COPD, GOLD stage 4, shows which of the following physical examination findings would the therapist expect to find?
A. Kyphosis with an increased thoracic excursion
B. Barreled chest with a decreased thoracic excursion
C. Pectus excavatum with an increased thoracic excursion
D. Pectus carinatum with decreased thoracic excursion

A

Correct Answer: B
A patient with severe COPD (GOLD 4) will have lost much of the elastic recoil properties of the lung. The usual elastic properties of the lung tissue help to pull the thorax into the normal chest wall configuration of health. Without these elastic recoil properties the patient’s thorax will “barrel” in appearance, meaning it is larger and rounder than what you would normally expect. As the thorax has moved into an inspiratory position at rest, there is less movement available, so a decreased thoracic excursion would be expected.

How well did you know this?
1
Not at all
2
3
4
5
Perfectly
59
Q

59.A patient is recovering from surgical resection of an acoustic neuroma and presents with symptoms of dizziness, vertigo, horizontal nystagmus, and postural instability. To address these problems, what should the physical therapy plan of care incorporate?
A. Repetition of movements and positions that provoke dizziness and vertigo
B. Hallpike’s exercises to improve speed in movement transitions
C. Strengthening exercises focusing on spinal extensors
D. Prolonged bedrest to allow vestibular recovery to occur

A

Correct Answer: A
In patients with unilateral vestibular pathology, habituation training (use of positions and movements that evoke symptoms) will encourage the vestibular system to recalibrate, Good recovery can generally be expected with gradual progression of exercises.

How well did you know this?
1
Not at all
2
3
4
5
Perfectly
60
Q

60.A physical therapist is treating a patient with diabetic peripheral neuropathy. The patient recently began taking Lyrica (pregabalin). During a monofilament exam of the feet the therapist notices circumferential marks bilaterally at the level of the malleoli after the socks are removed. The patient is complaining of increased difficulty ambulating long distances. In this situation, what is the therapist’s BEST course of action?
A. Contact the physician about possible development of congestive heart failure
B. Begin manual lymphatic drainage for secondary lymphedema
C. Complete the examination and instruct in proper skin care precautions
D. Educate the patient about the risks of foot ulceration

A

Correct Answer: A
It is important to recognize serious side effects of commonly used medications and to institute contact with the physician as appropriate. Lyrica is used to help treat diabetic neuropathy.
Serious side effects include heart failure, greater difficulty walking long-distances, and ymphedema (marks from the sock).

How well did you know this?
1
Not at all
2
3
4
5
Perfectly
61
Q

61.A patient presents to the clinic with pain and decreased function of the right shoulder. A full tear of the right rotator cuff musculature is suspected. The special test that would provide the most valid and reliable information confirming this suspicion would be the:
A. Drop arm test
B. Yergason’s test
C. Neer’s impingement test
D. Clunk test

A

Correct Answer: A
The primary intent of the drop arm test is to assess if the rotator cuff is intact. If the rotator cuff is fully torn, the drop arm test will be positive, meaning that the arm will fall from a fully elevated position if it is unsupported. the empty-can test is also a test for the rotator cuff muscles but is more specific for the supraspinatus.

How well did you know this?
1
Not at all
2
3
4
5
Perfectly
62
Q

62.A patient who is 5 weeks’ post myocardial infarction (MI) is participating in a cardiac rehabilitation program. The therapist is monitoring responses to increasing exercise intensity. The indicator that exercise should be immediately terminated is:
A. 1.5 mm of downsloping ST segment depression
B. Peak exercise HR > 140
C. Appearance of a PVC on the electrocardiogram (ECG)
D. Systolic BP > 140 mm Hg or diastolic BP > 80 mm Hg

A

Correct Answer: A
The upper limit for exercise intensity prescribed for patients post-MI is based on signs and symptoms. Of the choices, only ST segment depression (>1.0 mm of horizontal or downsloping depression) is a significant finding, representative of myocardial ischemia.

How well did you know this?
1
Not at all
2
3
4
5
Perfectly
63
Q

63.An infant who was 39 weeks gestational age at birth and is now 3 weeks chronological age demonstrates colic. In this case, what is the BEST intervention the PT should teach the mother?
A. Stroking and tapping
B. Neutral warmth
C. Visual stimulation with a colored object
D. Fast vestibular stimulation

A

Correct Answer: B
Neutral warmth achieved through wrapping or bundling the infant is a calming stimulus.

How well did you know this?
1
Not at all
2
3
4
5
Perfectly
64
Q

64.A therapist is treating a patient with Brown-Sequard syndrome that resulted from a gunshot wound. The therapist’s examination should reveal:
A. Sparing of tracts to sacral segments with preservation of perianal sensation and active toe flexion
B. Loss of motor function and pain and temperature sensation, with preservation of light touch and proprioception below the level of the lesion
C. Loss of motor function below the level of the lesion, primarily in the upper extremities
D. Ipsilateral loss of motor function, ipsilateral loss of light touch and proprioception, and contralateral loss of pain and temperature

A

Correct Answer: D
Brown-Sequard syndrome is a hemisection of the spinal cord characterized by ipsilateral loss of dorsal columns with loss of touch, pressure, vibration and proprioception; ipsilateral loss of Corticospinal tracts with loss of motor function below level of lesion; contralateral loss of spinothalamic tract with loss of pain and temperature below level of lesion; at lesion level bilateral loss of pain and temperature.

How well did you know this?
1
Not at all
2
3
4
5
Perfectly
65
Q

65.A patient is recovering from stroke and presents with moderate impairments of the left upper and lower extremities. The PT’s goal today is to instruct the patient in a stand-pivot transfer to the more affected side so the patient can go home on a weekend pass. The spouse is attending today’s session and will be assisting the patient on the weekend. What is the BEST choices for teaching this task?
A. Practice the task first with the patient then with the caregiver
B. Demonstrate the task, then have the caregiver practice with the patient
C. Practice the task first with the caregiver, then with the patient
D. Demonstrate the task, and then practice with the patient

A

Correct Answer: D
To ensure optimal motor learning, first demonstrate the task at ideal performance speeds. This provides the patient with an appropriate reference of correction (cognitive map) of the task.
Then use guided practice with the patient to ensure safety and successful performance.

How well did you know this?
1
Not at all
2
3
4
5
Perfectly
66
Q

66.A patient presents with low back pain of insidious onset. Based on the history and subjective complaints, the patient appears to have a dysfunction of a lumbar facet joint. What clinical test should be utilized to confirm this diagnosis?
A. McKenzie’s side glide test
B. Stork standing test
C. Slump test
D. Lumbar quadrant test

A

Correct Answer: D
The motion of the lumbar quadrant test places the lumbar facet joint in its maximally closed and therefore most provocative position, so if positive it is typically indicative of a lumbar facet dysfunction.

How well did you know this?
1
Not at all
2
3
4
5
Perfectly
67
Q
  1. When using a patellar tendon-bearing (PTB) prosthesis, a patient will experience excessive knee flexion in early stance if the;
    A. Socket is aligned too far back or tilted posteriorly
    B. Foot position is inset too much
    C. Socket is aligned too far forward or tilted anteriorly
    D. Foot position is outset too much
A

Correct Answer: C
In a PTB prosthesis, the socket is normally aligned in slight flexion to enhance loading on the patellar tendon, prevent genu recurvatum and resist the tendency of the amputated limb to slide too deeply into the socket. If it is aligned incorrectly (too far anterior or excessively flexed), it will result in excessive knee flexion.

How well did you know this?
1
Not at all
2
3
4
5
Perfectly
68
Q

68.A patient presents with Bluish discoloration of the skin and nail beds of fingers and toes.
Palms are also cold and moist. What is the MOST likely cause of these changes?
A. Carotenemia
B. Hypothyroidism
C. Cyanosis
D. Liver disease

A

Correct Answer: C
Bluish discoloration of the skin and nailbeds of fingers and toes, along with palms that are cold and moist, is indicative of cyanosis. It is caused by an excess of deoxygenated haemoglobin in the blood. It may be central (due to advanced lung disease, congenital heart disease, abnormal haemoglobin) or peripheral (decreased blood flow, venous obstruction).

How well did you know this?
1
Not at all
2
3
4
5
Perfectly
69
Q

69.An outpatient physical therapist is examining a patient who underwent a total knee arthroplasty 2 weeks ago. The patient reports that the entire leg has started swelling in the past 2 days. On examination there is pitting edema throughout the lower leg and foot with tenderness throughout the mid calf. Girth measurements reveal a 3.5 cm increase in the size of the mid calf in the symptomatic leg. What recommendation should the physical therapist make to the patient?
A. Rest, ice, and elevate the affected lower extremity
B. Go home and monitor symptoms. Phone the physician if there is no improvement in 24 hours
C. Go immediately to the emergency department
D. Go to the physician’s office after the therapy session for further assessment

A

Correct Answer: C
The patient scores a 3 on Well’s criteria for deep vein thrombosis, placing him/herin the high probability category. The most appropriate response for the physical therapist would be to send the patient to the emergency department for further assessment.

How well did you know this?
1
Not at all
2
3
4
5
Perfectly
70
Q

70.A patient complains of persistent wrist pain after painting a house 3 weeks ago. The patient demonstrates signs and symptoms consistent with de Quervain’s tenosynovitis. An appropriate special test to confirm the diagnosis is:
A. Finkelstein’s test
B. Phalen’s test
C. Froment’s sign
D. Craig’s test

A

Correct Answer: A
Finkelstein’s test is specific for reproducing the pain associated with de Quervain’s tenosynovitis of the abductor pollicis longus and extensor pollicis brevis.

How well did you know this?
1
Not at all
2
3
4
5
Perfectly
71
Q

71.As part of the chart review, the physical therapist views the patient’s most current chest film. Based on this film, what is the MOST likely examination finding?
A. Increased lateral costal expansion
B. Increased subcostal angle
C. Decreased inspiration: expiration (I:E) ratio
D. Decreased mediate percussion

A

Correct Answer: B
This film demonstrates a patient with hyperinflated lungs as evidenced by the flattened diaphragm, blunted costophrenic angle, and increased amount of air. This will cause the subcostal angle to increase significantly.

How well did you know this?
1
Not at all
2
3
4
5
Perfectly
72
Q

72.An elderly patient with persistent balance difficulty and a history of recent falls (two in the past 3 months) is referred for physical therapy examination and evaluation. During the initial examination, what should the therapist examine first?
A. Level of dyspnea during functional transfers
B. Cardiovascular endurance during a 6-minute walking test
C. Sensory losses and sensory organization of balance
D. Spinal musculoskeletal changes secondary to degenerative joint disease (DJD)

A

Correct Answer: C
A critical component of balance control is sensory input from somatosensory, visual and vestibular receptors, and overall sensory organization of inputs. Initial examination should address these elements before moving on to assess the motor components of balance (e.g., postural synergies). The Clinical Test for Sensory Integration in Balance (CTSIB) or modified CTSIB (Shumway-Cook, Horak) are appropriate instruments.

How well did you know this?
1
Not at all
2
3
4
5
Perfectly
73
Q

73.To reduce an elderly individual’s chronic forward head posturing in standing and sitting the therapist should consider stretching exercises to:
A. Middle trapezius and rhomboid muscles
B. Rectus capitis anterior muscles
C. Longus capitis and longs colli muscles
D. Rectus capitis posterior minor and rectus capitis posterior major muscles

A

Correct Answer: D
Forward head posturing or forward translation of the occiput in relation to the neck and trunk is associated with extension of the occipital axial joint and flexion of the lower and mid cervical spines. Chronic extension of the occipital axial joint will lead to shortening of the suboccipital extensor muscles (rectus capitis posterior major and minor), and localized stretching of these muscles would be indicated as part of a therapeutic intervention to reduce forward head posturing.

How well did you know this?
1
Not at all
2
3
4
5
Perfectly
74
Q
  1. What are the major benefits of using the 6-Minute Walking Test as an outcome measure?
    A. Accurately documents maximal exercise capacity
    B. Provides good correlation with functional abilities
    C. Allows determination of severity of lung disease
    D. Provides determination of peak oxygen uptake
A

Correct Answer: B
The 6-Minute Walk Test (6MWT) shows a good correlation with function, as the 6MWT is a submax test, and function is performed at a submax work level.

How well did you know this?
1
Not at all
2
3
4
5
Perfectly
75
Q

75.During a postural screen for a patient complaining of low back pain, the therapist notices that the knees are in genu recurvatum, What are the common contributory problems for which the therapist should examine?
A. Ankle dorsiflexion and hip abduction
B. Forefoot varus and posterior pelvic tilt
C. Ankle plantarflexion and anterior pelvic tilt
D. Lateral tibial torsion and anterior pelvic tilt

A

Correct Answer: C
A common contributory problem or correlated motion for genu recurvatum is ankle plantarflexion due to shortened gastrocnemius muscles. Alterations occurring up the kinetic chain include anterior pelvic tilt to maintain the center of gravity over the feet.

How well did you know this?
1
Not at all
2
3
4
5
Perfectly
76
Q

76.After treating a patient for trochanteric bursitis for 1 week, the patient has no resolution of pain and is complaining of problems with gait. After re-examination, the therapist finds weakness of the quadriceps femoris and altered sensation at the greater trochanter. What is the MOST likely cause of the problems?
A. L5 nerve root compression
B. Sacroiliac (SI) dysfunction
C. L4 nerve root compression
D. Degenerative joint disease (DJD) of the hip

A

Correct Answer: C
The positive findings are consistent with an L4 nerve root compression.

How well did you know this?
1
Not at all
2
3
4
5
Perfectly
77
Q

77.A patient in chronic renal failure is being seen in physical therapy for deconditioning and decreased gait endurance. The therapist needs to schedule the patient’s sessions around dialysis, which is received three mornings a week. What guidelines should the therapist follow when taking the patient’s blood pressure?
A. Every minute during walking, using the nonshunt arm
B. Pre-and postactivities, using the nonshunt arm
C. In sitting when activity has ceased, using the shunt arm
D. In the supine position, using the shunt arm

A

Correct Answer: B
A dialysis shunt would interfere with taking BP. Using the nonshunt arm. Pre- and postexercise measurements are appropriate.

How well did you know this?
1
Not at all
2
3
4
5
Perfectly
78
Q

78.A patient who is currently being treated for low back pain arrives for therapy complaining of pain across the middle of the right chest and back, When the therapist inspects the skin, clustered vesicles are apparent in a linear arc. The surrounding skin is hypersensitive. What is the MOST likely diagnosis?
A. Herpes simplex infection
B. Psoriasis
C. Dermatitis
D. Herpes zoster infection

A

Correct Answer: D
Herpes zoster is an acute infection caused by reactivation of the latent varicella-zoster virus (shingles). It is characterized by painful vesicular skin eruptions that follow the underlying route of a spinal (in this case) or cranial nerve. Additional symptoms include fever, gastrointestinal disturbances, malaise, and headache.

How well did you know this?
1
Not at all
2
3
4
5
Perfectly
79
Q

79.While providing sports coverage at a local high school, a physical therapist is asked to examine an athlete with a knee injury, Based on the mechanism of injury, the therapist suspects rupture of the ACL. What test should be performed immediately to identify a torn ACL?
A. McMurray’s test
B. Reverse Lachman’s stress test
C. Lachman stress test
D. Posterior sag test

A

Correct Answer: C
Lachman’s stress test is the primary clinical test utilized to identify if the anterior cruciate ligament is intact or ruptured.

How well did you know this?
1
Not at all
2
3
4
5
Perfectly
80
Q
  1. After a myocardial infarction (MI), a patient is a new admission to a phase 3 hospital-based
    Cardiac rehabilitation program. During the initial exercise session, the patient’s ECG responses are continuously monitored via radio telemetry. The therapist notices three PVCs occurring in a run with no P wave. The therapist should:
    A. Modify the exercise prescription by decreasing the intensity
    B. Stop the exercise and notify the physician immediately
    C. Continue the exercise session, but monitor closely
    D. Have the patient sit down and rest for a few minutes before resuming exercise
A

Correct Answer: B
A run of three or more PVCs occurring sequentially is ventricular tachycardia. The rate is very rapid, resulting in seriously compromised cardiac output. This is potentially an emergency situation that can deteriorate rapidly into ventricular fibrillation (no cardiac output) and cardiac arrest.

How well did you know this?
1
Not at all
2
3
4
5
Perfectly
81
Q

81.A patient with left hemiplegia is able to recognize his wife after shie is with him for a while and talks to him, but is unable to recognize the faces of his children when they come to visit.
The children are naturally very upset by their father’s behavior. The BEST explanation for his problem is:
A. Somatognosia
B. Anosognosia
C. Visual agnosia
D. ideational apraxia

A

Correct Answer: C
All of the choices are indicative of perceptual dysfunction. This patient is most likely suffering from visual agnosia, which is an inability to recognize familiar objects despite normal function of the eyes and optic tracts. Once the wefe talks with him, he is able to recognize her by her
voice.

82
Q

82.A home care PT receives a referral to evaluate the fall risk potential of an elderly community-dweller with chronic coronary artery disease (CAD). The patient has fallen three times in the past 4 months, with no history of fall injury except for minor bruising. The patient is currently taking a number of medications. What is the drug that is MOST likely to contribute to dizziness and increased fall risk?
A. Colace
B. Albuterol
C. Nitroglycerin
D. Coumadin sodium

A

Correct Answer: C
Of the medications listed, nitroglycerin has the greatest risk of causing dizziness or weakness due to postural hypotension. Fall risk is increased even with small doses of nitroglycerin.

83
Q

83.A patient complains of foot pain when first arising that eases with ambulation. The therapist finds that symptoms can be reproduced in weight bearing and running on a treadmill.
Examination reveals pes plans and pain with palpation at the distal aspect of the calcaneus.
Early management would include:
A. Prescription for a customized orthosis
B. Strengthening of ankle dorsiflexors
C. Modalities to reduce pain
D. Use of a resting splint at night

A

Correct Answer: D
The symptoms are suggestive of plantar fasciitis. The focus of patient management should be on decreasing the irritation to the plantar fascia. This is most effectively done with a resting night splint.

84
Q

84.A patient with a T10 paraplegia (ASIA A) resulting froma spinal cord injury is ready to begin community wheelchair training. The therapist’s goal is to teach the patient how to do a wheelie in order to manage curbs. What is the BEST training strategy to instruct the patient in performing a wheelie?
A. Place a hand on the top of the handrims to steady the chair while throwing the head and trunk forward.
B. Throw the head and trunk backward to rise up on the large wheels
C. Lean backward while moving the hands slowly backward on the rims
D. Grasp the handrims posteriorly, and pull them forward abruptly and forcefully

A

Correct Answer: D
A wheelie can be assumed by having the patient place his/her hands posterior on the handrims and pulling them abruptly and sharply forward. If the patient is unable to lift the casters in this manner, throwing the head back forcefully when pulling the handrims may work. An alternate technique is to grasp the handrims anteriorly, pull backward, then abruptly and forcefully reverse the direction of pull. The therapist can assist by steadying the chair at the patient’s balance point until the patient learns to adjust the position through the use of handrim movements forward and backward.

85
Q

85.A PT should be aleit to recognize the signs and symptoms associated with the onset of aspiration pneumonia. Which patient diagnosis is the MOST susceptible to develop this from of pneumonia?
A. A circumferential burn of the thorax associated with significant pain
B. Severe scoliosis with compression of internal organs, including the lungs
C. Amyotrophic lateral sclerosis (ALS) with dysphagia and diminished gag reflex
D. A complete spinal cord lesion at T2 with diminished coughing ability and forced vital capacity (FVC)

A

Correct Answer: C
(Aspiration pneumonia results from an abnormal entry of fluids or matter (including food) into the airways. A patient with ALS with an inability to swallow (dysphagia) and diminished gag reflex is most susceptible to aspiration pneumonia.

86
Q

86.A patient demonstrates quadriceps weakness (4/5) and difficulty descending stairs. The BEST intervention to regain functional strength in the quadriceps is:
A. Progressive resistance exercises, 70% 1 repetition maximum, three sets of 10
B. Partial squats, progressing to lunges
C. Maximum isometric exercise, at 45° and 90° of knee extension
D. Isokinetic exercise, at 36°sec

A

Correct Answer: B
Closed-chain exercises are the most appropriate in this example because of the patient’s difficulty descending stairs. Moving the body over a fixed distal segment provides loading to muscles, joints, and noncontractile soft tissues while stimulating the sensory receptors needed for stability and balance.

87
Q

87.Independent community ambulation as the primary means of functional mobility is a realistic functional expectation for a patient with the highest level of complete spinal cord injury (ASIA A) at:
A. Low lumbar (L4-5)
B. Low thoracic (T9-10)
C. Midthoracic (T6-9)
D. High lumbar (T12-L1)

A

Correct Answer: A
Patients with low lumbar lesions (L4-5) can become independent and functional ambulators providing that they exhibit adequate ROM< strength of residual muscles and cardiovascular endurance. The locomotor training program emphasizes learning to use bilateral ankle-foot orthoses (AFOs) and canes.

88
Q

88.A patient presents with multiple fractures of both hands and wrists as a result of a mountain bike accident. Now, 5 weeks later, the patient has limited wrist and finger motion and dry, scaly skin over the involved areas. What is the biophysical agent that would provide the GREATEST benefit?
A. Contact ultrasound (US)
B. Hot packs
C. Paraffin
D. Functional electrical stimulation

A

Correct Answer: C
Paraffin bath will provide circumferential heating of the hands and fingers and aid in softening the skin prior to exercise.

89
Q

89.Anelderly patient demonstrates a history of recent falls (two in the past 2 months) and mild balance instability. The therapist’s referral is to examine the patient and recommended an assistive device as needed. Based on the patient’s history, it would be BEST to select a:
A. Folding reciprocal walker
B. Standard, fixed-frame walker
C. Front wheel rolling walker that folds
D. Hemi walker

A

Correct Answer: C
A rolling walker will provide added stability, while maintaining gait as a continuous movement sequence. The additional benefit of a folding walker facilitates easy transport and mobility in the community.

90
Q

90.After surgery, a patient develops a stiff pelvis and limited pelvic/lower trunk mobility. The therapist elects to use sitting exercises on a therapy ball to correct these impairments, In order to improve lower abdominal control, what direction should the patient move the ball?
A. Backward, producing anterior tilting of the pelvis
B. Forward, producing posterior tilting of the pelvis
C. Forward, producing anterior tilting of the pelvis
D. Backward, producing posterior tilting of the pelvis

A

Correct Answer: B
Contraction of the lower abdominals results in posterior tilting of the pelvis and can be achieved with forward or anterior movement of the therapy ball.

91
Q

91.A patient presents with weakness in the right lower leg 3 weeks after a motor vehicle accident. The patient complains of spontaneous twitching in the muscles of the lower leg.
The therapist visually inspects both limbs and determines that muscle bulk is reduced on the involved right limb, Girth measurements confirm a 1-inch difference in the circumference of the right leg measured 4 inches below the patella. Deep tendon reflexes and tone are diminished. What is the MOST likely cause of the patient’s weakness?
A. Peripheral nerve injury
B. Pyramidal tract dysfunction in the medulla
C. Guillan-Barre syndrome
D. Brainstem dysfunction affecting extrapyramidal pathways

A

Correct Answer: A
This patient is exhibiting signs and symptoms of lower motor neuron injury (hypotonia, hyporeflexia, paresis, neurogenic atrophy). The presence of muscle fasciculations is a hallmark sign of lower motor injury.

92
Q

92.Anelderly adult patient presents with a history of and subjective complaints consistent with lumbar central spinal stenosis, The most appropriate clinical test used to differentiate spinal stenosis from intermittent vascular claudication would be:
A. Femoral nerve traction test
B. Bicycle (van Gelderen’s) test
C. Valsalva’s maneuver
D. Lumbar quadrant test

A

Correct Answer: B
The bicycle (van Gelderen’s) test is designed to differentiate between spinal stenosis and intermittent vascular claudication. Van Gelderen’s bicycle test is designed to stress the LE vascular system without causing any central canal or foraminal stenosis that could be misinterpreted as intermittent neurogenic claudication.

93
Q

93.A patient with bacterial pneumonia has crackles and wheezes in the left lateral basal segment and decreased breath sounds throughout. The patient is on 4 L of oxygen by nasal cannula with a resulting arterial oxygen saturation (SaOz) of 90%. Respiratory rate is 28. What is the (MOST BENEFICIAL intervention for this case?
A. Postural drainage, percussion, and shaking over the appropriate area on the left lateral thorax for secretion removal
B. Positioning in left side-lying to improve ventilation/perfusion ratios
C. Postural drainage, percussion, and shaking to the right basilar segments in order to keep the right lung healthy
D. Breathing exercise encouraging expansion of the right lateral basilar thorax, because the left side is not currently participating in gas exchange

A

Correct Answer: A
A treatment of postural drainage, percussion, and shaking to the appropriate lung segments is advisable. The standard postural drainage position for the lateral basilar segment of the left lower lobe is in side-lying position with the head of bed tipped in full Trendelenburg position.
Given the borderline SaO, values on 4 L of oxygen, modification of the position may be necessary for patient tolerance.

94
Q

94.A therapist receives a referral to see an elderly patient in the intensive care unit (ICU)
recovering from a severe case of pneumonia. The therapist recognizes that the disorientation is due to delirium rather than dementia because the:
A. Patient has hallucinations throughout the day
B. Patient demonstrates persistent personality changes
C. Symptoms are intermittent
D. Level of arousal is significantly depressed

A

Correct Answer: C
cutely ill, hospitalized elderly patients frequently exhibit delirium, a fluctuating attention state. Patients demonstrate a fluctuating course with symptoms of confusion that alternate with lucid intervals. Sleep/wake cycles are disrupted and confusion is typically worse at night.

95
Q

95.A comparison of the effects of exercise in water, on land, or combined on the rehabilitation outcome of patients with intra-articular anterior cruciate ligament reconstructions revealed that less joint effusion was noted after 8 weeks in the water group. An appropriate statistical test to compare the girth measurements of the three groups is:
A. Analysis of covariance
B. Spearman’s rho
C. Chi-square
D. ANOVA

A

Correct Answer: D
NOVA is a parametric statistical test used to compare three or more treatment groups (in this example, in water, on land, or combined exercise groups) on a measure of the dependent variable (joint effusion girth measurements) at a selected probability level.

96
Q

96.A patient has a recent history of strokes (two in the past 4 months) and demonstrates good return in the right lower extremity, The therapist is concentrating on improving balance and independence in gait. Unfortunately, speech recovery is lagging behind motor recovery. The patient demonstrates a severe fluent aphasia. What is the BEST strategy to use during physical therapy sessions?
A. Demonstrate and gesture to get the idea of the task across
B. Have the family present to help interpret during physical therapy sessions
C. Utilized verbal cues, emphasizing consistency and repetition
D. Consult with the speech pathologist to establish a communication board

A

Correct Answer: A
Fluent aphasia (Wernicke’s aphasia) is a central language disorder in which spontaneous speech is preserved and flows smoothly while auditory comprehension is impaired. Demonstration and gesture (visual modalities) offer the best means of communicating with this patient since the impairment is severe.

97
Q

97.During a gait examination, a patient with a transfemoral prosthesis demonstrates terminal swing impact. Additional examination is needed as the therapist suspects the:
A. Prosthesis has too little tension in the extension aid
B. Hip flexors are weak
C. Prosthesis is externally rotated
D. Prosthesis has insufficient knee friction

A

Correct Answer: D
Terminal swing impact refers to the sudden stopping of the prosthesis as the knee extends during late swing, Possible causes can include insufficient knee friction or too much tension in the extension aid. In addition, if the patient with an amputation fears the knee will buckle at heel strike, the patient can use forceful hip flexion to extend the knee.

98
Q

98.A patient was burned over 40% of the body in an industrial accident and has full-thickness burns over the anterior trunk and neck and superficial partial-thickness burns over the shoulders. In order to stabilize this patient out of positions of common deformity, what orthotic device would be of GREATEST benefit?
A. Soft cervical collar with an intrinsic plus hand splint
B. A cervical thoracic lumbosacral orthosis (CTLSO) used during all upright activities
C. Plastic cervical orthosis and axillary splints utilizing an airplane position
D. Splints utilizing a flexed position for the shoulders and body jacket for the trunk

A

Correct Answer: C
The common deformity for the anterior neck is flexion; the appropriate positioning device is a firm rigid plastic cervical collar that stresses extension. The common deformity of the shoulders in adduction and internal rotation; the appropriate position device is an axillary or airplane splint that stresses abduction, flexion, and external rotation.

99
Q

99.A patient with type 2 diabetes is referred to physical therapy for exercise conditioning. What is a pathophysiologic cause of type 2 diabetes?
A. Metabolic syndrome
B. Impaired ability of the tissues to use insulin and insulin deficiency
C. Loss of beta-cell function and insulin deficiency
D. Pancreatic tumor

A

Correct Answer: B
Type 2 diabetes results from impaired ability of the tissues to use insulin (insulin resistance), accompanied by a relative lack of insulin or impaired release of insulin.

100
Q
  1. Which intervention is BEST to improve left-sided neglect in a patient with left hemiplegia?
    A. Hook-lying, holding, light resistance to both hip abductors
    B. Rolling, supine to side-lying on right, using a PNF lift pattern
    C. Sitting, with both arms extended, hands resting on support surface, active holding
    D. Bridging with both arms positioned in extension at the sides
A

Correct Answer: B
Incorporating the involved left side into a crossing the midline activity (rolling, using PNE lift) is best.

101
Q
  1. patient with no significant past medical history who now presents with a bacterial pneumonia in the right anterior base would present with which of the following exam findings?
    A. Decreased breath sounds throughout all lung fields, increased SaO2, febrile
    B. Bronchial breath sounds at the right anterior base, increased Sa2, febrile
    C. Crackles on inspiration only at right anterior base, decreased SaO2 and productive cough x 3 days
    D. Wheezes on inspiration only throughout the right lung fields, decreased SaO2, dry cough × 1 day.
A

Correct Answer: C
Bacterial pneumonia has a gradual onset of days with a productive cough. As pneumonia interferes with the transport of oxygen from the alveoli to the pulmonary capillaries, the Pa0, and therefore the Sal would be lower than expected, and crackles in the area of the pneumonia is a usual finding.

102
Q
  1. A patient with chronic obstructive pulmonary disease (COPD reports to the fourth outpatient pulmonary rehabilitation session complaining of nausea, gastric upset, and feeling jittery. The patient reports no change in pulmonary symptoms. The PT records the following set of vital signs: temperature 98.6°F. HR 110 beats/min and irregular, BP 150/86, respiratory rate 20. Breath sounds show no change from baseline. The therapist checks the medical record and finds that the patient has no history of gastric disease. The patient is presently taking theophylline, albuterol sulfate (ventolin), and triamcinolone diacetate (Amcort). What action should the PT take?
    A. Call the patient’s physician immediately and report signs of theophylline toxicity
    B. Have the patient increase use of Ventolin to improve respiratory status
    C. Have the patient stop use of Amcort until he/she schedules an appointment with the physician
    D. Send the patient home and notify the physician of current symptoms
A

Correct Answer: A
Theophylline is a bronchodilator used to reverse airway obstruction. The combination of symptoms of irregular HR, feeling jittery, and gastric upset is consistent with theophylline toxicity, Because theophylline toxicity can cause arrhythmias and seizures, the patient’s physician should be notified by the PT rather than wait for the patient to return home to call the physician. It is also likely that a blood test will be needed to check the theophylline toxicity level and this could be done at the facility.

103
Q
  1. The therapist is treating a 1-year-old child with Down syndrome at home and notices decreasing strength in the extremities, with neck pain and limited neck motion. Upper extremity deep tendon reflexes (DTRs) are 3+, These signs and symptoms are a hallmark of what diagnosis?
    A. Lower motor neuron signs consistent with Down syndrome
    B. Atlanto-axial subluxation with lemniscal impingement
    C. Upper motor neuron signs consistent with Down syndrome
    D. Atlanto-axial subluxation with spinal cord impingement
A

Correct Answer: D
Ligamentous laxity is a hallmark of Down syndrome and can lead to atlanto-axial instability
(AAI) with spinal cord impingement. This is a medical emergency situation. Decreased muscle strength and increased DTRs are the signs of dislocation from loss of cord function. In this case, the increasing symptomatology (changes in strength, neck pain, limited neck motion, hyperreflexia) is significant for a developing subluxation. Other LMN signs (clonus, positive Babinski response) may also occur.

104
Q
  1. Which of the following factors is likely to contribute to subluxation and shoulder pain in hemiplegia?
    A. Spastic paralysis of the biceps
    B. Traction using on a depressed, downwardly rotated scapula
    C. PROM with normal scapulohumeral rhythm
    D. Spastic retraction with elevation of scapula
A

Correct Answer: B
Shoulder pain and subluxation in hemiplegia may be caused by a number of different factors.
One major causes is traction and gravitational forces acting on a depressed, downwardly rotated scapula.

105
Q
  1. The therapist tests a patient’s hearing by holding a vibrating tuning fork on the mastoid process and then in front of the ear. Comparison is made by asking the patient which is louder. What is the name of this test?
    A. Weber’s test
    B. Rinne’s test
    C. Hallpike-Dix test
    D. Caloric test
A

Correct Answer: B
Rinne’s test is used to compare bone conduction (BC) with air conduction (AC). If BC is greater than AC, the patient is experiencing conductive deafness, If AC is greater than BC, the patient is experiencing sensorineural deafness.

106
Q
  1. A patient with hemiplegia and a drop foot is referred for physical therapy gait training.
    Examination reveals a pressure ulcer on the patient’s right heel (pictured). The ulcer has dry eschar without edema, erythema, fluctuance, or drainage. The patient is febrile. What is the BEST choices for intervention?
    A. Sharp debridement
    B. Refer for an arterial bypass graft
    C. Use an AFO with heel pressure relief
    D. Enzymatic debridement
A

Correct Answer: C
The AFO helps to prevent plantarflexion contractures, while the heel pressure relief prevents further damage to the heel and promotes healing.

107
Q
  1. A patient presents with an acute sprain of the right ankle. According to the patient, this has occurred fairly frequently over the past 5 years. What clinical test should the therapist use to examine the integrity of the anterior talofibular ligament?
    A. Anterior drawer test
    B. Morton’s test
    C. Talar tilt
    D. Thompson’s test
A

Correct Answer: A
The anterior drawer test is specifically designed to assess the integrity of the anterior talofibular ligament.

108
Q
  1. A patient with left knee degenerative joint disease (DJD) complains of left-sided knee pain of 2 months’ duration, The patient has been followed by outpatient physical therapy for 3 weeks and feels this condition is worsening. Pain has increased during weight-bearing activities, and the patient can no longer fully extend the left knee, Examination findings include increased swelling, decreased knee AROM into extension, and an antalgic gait.
    What action should the PT take?
    A. Tell the patient to see an orthopedic surgeon for possible immediate surgical intervention
    B. Immediately return the patient to the referring physician with documentation indicating that treatment was ineffective.
    C. Continue physical therapy for another 2 weeks, because there is uncertainty whether the patient understands or is complying with the home exercise program.
    D. Continue therapy for another week to ensure that all interventions have been attempted, and then return the patient to the referring physician.
A

Correct Answer: B
In this case, it would be best to send the patient back to the referring physician with an explanation of what was done, the ineffectiveness of the treatment, and any suggestions for further follow-up.

109
Q
  1. A postpartum patient with stress incontinence is referred for physical therapy. The therapist instructs the patient in pelvic floor exercises. What is the BEST choice for initial exercise?
    A. Supine, squeeze the sphincters and hold for 10 seconds
    B. Hooklying, bridge and hold for 5 seconds
    C. Sitting on toilet, stop and hold the flow of urine for 5 seconds during urination
    D. Supine, squeeze the sphincters and hold for 3 seconds.
A

Correct Answer: C
The stop exercise consists of sitting on the toilet and stopping and holding the flow of urine for 5 seconds before starting the flow again. This helps the patient identify the pelvic floor muscles.
Once these muscles are identified, exercise can be progressed to tightening the muscles for 5 seconds and relaxing for 5 seconds with an empty bladder. Times are increased to 10 seconds each phase. The patient is instructed to repeat 3 sets of 10 at least 3 times a day.

110
Q
  1. A patient with stroke demonstrates early recovery in the right upper extremity (RUE) with moderate spasticity in the biceps and finger flexors. Voluntary movement is evident in elbow flexors and shoulder abductors only (through ½ range). What is the BEST choices for initial exercise?
    A. Functional activities emphasizing ADL using the less-affected UE
    B. Facilitation of early movements in the flexor synergy pattern
    C. Weight bearing on an extended RUE with wrist and fingers extended
    D. Prolonged positioning of the RUE in a hemisling.
A

Correct Answer: C
Early intervention should focus on stretching and positioning the RUE into extension with the wrist and fingers extended. This helps decrease the developing flexor spasticity. Weight bearing on the limb promotes extensor activity in the triceps and shoulder stabilizers.

111
Q

A physical therapist and a physician are at odds regarding ordering a power wheelchair for a 3-year-old child. What is the MOST important factor to consider for not recommending a power wheelchair for this child?
A. Age of the child
B. Child has quadriplegic cerebral palsy
C. Child is non-verbal
D. Child has poor head and fine motor control

A

Correct Answer: D
Most power wheelchairs require good head control to use a head rest control system or good fine motor skills to use a joy stick. Although there are other ways to propel a power chair, this is the best reason to NOT recommend a power chair.

112
Q

An ambulatory patient recovering from a left CVA is wearing a plastic AFO to stabilize the right foot. During gait analysis, the therapist observes lateral trunk bending toward the right as the patient bears weight on the right leg at midstance. What is the BEST intervention to correct this problem?
A. Strengthen hip flexors on the right side
B. Provide a lift on the shoe of the involved leg
C. Strengthen the hip abductors on the right side
D. Strengthen hamstrings on the right side

A

Correct Answer: C
The lateral trunk bending (Trendelenburg gait) is the result of weak hip abductors on the right (a common problem for patient recovering from stroke. Strengthening of the abductors on the involved right side is indicated.

113
Q

A 10-year-old boy who play catcher on a baseball team complains of bilateral team complains of bilateral knee pain that is exacerbated with forceful quadriceps contraction.
Examination reveals pain and swelling at the distal attachment of the patellar tendon. What is the BEST early intervention to use?
A. Decreased loading of the knee by the quadriceps femoris muscle
B. Strengthening exercises for the quadriceps femoris to prevent disuse atrophy
C. Casting followed by decreased loading of the knee
D. Ultrasound to decrease pain and inflammation

A

Correct Answer: A
Baseball catchers must make forceful contractions of the quadriceps muscles each time they stand up to throw the ball to the pitcher, This may precipitate Ogood-Schlatter disease in the adolescent or preadolescent boy. Early intervention of this condition focuses on reduction of the loading by the quadriceps, but still retaining normal lower extremity function.

114
Q
  1. A patient presents with a decubitus ulcer of 3 months’ duration on the lateral ankle, The ankle is swollen, red, and painful, with a moderate to high amount of wound drainage (exudate). What is the BEST choice of dressing for this wound?
    A. Hydrogel dressings
    B. Semipermeable film dressings
    C. Calcium Alginate dressings
    D. Gauze dressings
A

Correct Answer: C
Wounds with moderate to high exudate benefit from calcium alginate dressings. The dressings absorb large amounts of exudate (up to 20 times their weight) and form a gel, which maintains the moist wound environment while maintaining good permeability to oxygen.

115
Q
  1. A patient was diagnosed with a bulging disc at the right L5-S1 spinal level without nerve root compression. What is the impairment MOST likely to be documented?
    A. Centralized gnawing pain with loss of postural control during lifting activities
    B. Centralized gnawing pain with uncompensated gluteus medius gait
    C. Radicular pain to the right great toe with a compensated gluteus medius gait
    D. Radicular pain to the right great toe with difficulty sitting for long periods
A

Correct Answer: A
Discal degeneration without nerve root compression would likely be exhibited as a centralized gnawing pain with loss of proprioception.

116
Q
  1. A patient recovering from traumatic brain injury (TBI) demonstrates impaired cognitive function (Rancho Cognitive Level VII), What training strategy should be the therapist’s focus.
    A. Provide assistance as needed using guided movements during training
    B. Provide a high degree of environmental structure to ensure correct performance
    C. Involve the patient in decision-making and monitor for safety
    D. Provide maximum supervision as needed to ensure successful performance and safety
A

Correct Answer: C
As patients with TBI recover, structure and guidance must be gradually reduced and patient involvement in decision-making increased. Safety must be maintained while increasing levels of independence are fostered. Patients at stage VII of recovery often exhibit rote movements (robot syndrome) indicative of the highly structured training utilized for patients during earlier stages of recovery.

117
Q
  1. What is the MOST appropriate intervention to correct for the problem of a forward festinating gait in a patient with Parkinson’s disease?
    A. Use of a heel wedge
    B. Use of a toe wedge
    C. Increase stride length using floor markers
    D. Increase cadence using a metronome
A

Correct Answer: B
A festinating gait is an abnormal and involuntary increase in the speed of walking in an attempt to catch up with a displaced center of gravity due to the patient’s forward lean. The most appropriate intervention would be to use a toe wedge, which would help to displace the patient’s center of gravity backward.

118
Q
  1. A patient presents with difficulty with fast movement speeds and fatigues easily. The therapist decides on a strength training program that specifically focuses on improving fast-twitch muscle fiber function, What is the optimal exercise prescription to achieve this goal?
    A. High-intensity workloads for short durations
    B. Low-intensity workloads for long durations
    C. Low-intensity workloads for short durations
    D. High-intensity workloads for long durations
A

Correct Answer: A
High-intensity exercises at fast contraction speeds for shorter durations (>20 repetitions) are needed to train the highly adaptable fast-twitch Ila fibers.

119
Q
  1. A cross-country runner presents with a complaint of pain in the proximal one-third of the right tibia with an insidious onset 4 weeks ago, The pain is present intermittently, and running exacerbates the symptoms. Ligamental testing and soft tissue examination of the knee and leg are unremarkable. Which imaging studies are recommended to be performed INITIALLY in order to help establish a diagnosis?
    A. Radiograph and if positive, then MRI
    B. CT scan and T2 MRI
    C. Bone scan and if positive, MRI
    D. Radiograph and if negative, a bone scan
A

Correct Answer: D
This case represents the typical presentation of a stress fracture. Plain radiographs are an inexpensive and quick method to diagnose fractures and healing rates. Stress fractures may take 2-8 weeks before they can be visualized on plain films. If the plain films were negative, then a bone scan is performed (stress fracture, bone bruising).

120
Q
  1. What is the MOST appropriate functional goal for a 5-year-old child with a high lumbar desion (myelomeningocele, L2 level) and minimal cognitive involvement?
    A. Household ambulation with KAFOs and rollator walker
    B. Community ambulation with an RGO and Lofstrand crutches
    C. Household ambulation with a reciprocating gait orthosis (RGO) and Lofstrant crutches
    D. Community ambulation with HKAFOs and Lofstrand crutches
A

Correct Answer: C
Achild with a high-level myelomeningocele will be able to ambulate for limited (household) distances with an RGO and Lofstrand crutches. Physiological benefits include improved cardiovascular and musculoskeletal functions.

121
Q
  1. A patient with traumatic onset (motor vehicle accident) of neck pain presents with subjective complaints of frank upper cervical spine instability. Which test would safely assist in identifying the integrity of the C1-2 articulation?
    A. Transverse ligament stress test
    B. Vertebral artery test
    C. Maximum cervical compression test
    D. Hautant’s test
A

Correct Answer: A
The transverse ligament stress test is specifically designed to assess the integrity of the transverse ligament, which maintains the positions of the dens of C2 with the anterior arch of C1.

122
Q
  1. The therapist is reading a recent report of arterial blood gas analysis with the following values:
    Fraction of inspired oxygen (Fi02) = 0.21
    Arterial oxygen pressure (Pa02) = 53 mm Hg
    Arterial carbon dioxide pressure (PaCO2) = 30 mm Hg
    pH = 7.48
    A. Metabolic alkalosis
    B. Respiratory alkalosis
    C. Metabolic acidosis
    D. Respiratory acidosis
A

Correct Answer: B
This arterial blood gas shows an increased pH, which is an alkalosis. When looking at arterial blood gas values, carbon dioxide can be viewed essentially as an acid. If the carbon dioxide level is low, then you have less acid, or a resulting alkalosis. This is, therefore, a respiratory alkalosis.

123
Q

Aretired patient is referred to a cardiac exercise group after a mild MI. From the intake questionnaire, the therapist learns the patient has type 1 insulin-dependent diabetes mellitus (IDDM), controlled with twice daily insulin injections. When initiating an exercise program, how should the therapist instruct the patient in order to minimize the risk of a hypoglycemic event?
A. Exercise daily for 40-50 minutes to achieve proper glucose control
B. Avoid exercise during periods of peak insulin activity
C. Monitor blood glucose levels carefully every week during the rehabilitation program
D. Decrease carbohydrate intake for 2 hours before the exercise session

A

Correct Answer: B
The patient should monitor blood glucose levels frequently when initiating an exercise program and avoid exercise during periods of peak insulin activity (2-4 hours after injection). The therapist should use RPE in addition to HR to monitor exercise intensity.

124
Q
  1. An elderly resident of a community nursing home is diagnosed with Alzheimer’s type dementia. What patient behaviors should the therapist consider when formulating the plan of care?
    A. Can reliably perform wheelchair transfer activities
    B. Is more likely to remember current experiences than past ones
    C. Will likely be resistant to activity training if unfamiliar activities are used
    D. Can usually be trusted to be responsible for own daily care needs
A

Correct Answer: C
Activity training is most likely to be successful if done with familiar activities

125
Q
  1. A patient with spinal cord injury is having difficulty learning how to transfer from mat to wheelchair. The patient just cannot seem to get the idea of how to coordinate this movement.
    In this case, what is the MOST effective use of feedback during early motor learning?
    A. Focus on knowledge of performance and proprioceptive inputs
    B. Focus on guided movement and proprioceptive inputs
    C. Provide feedback only after a brief (5-sec) delay
    D. Focus on knowledge of results and visual inputs
A

Correct Answer: D
During the early stage of motor learning (cognitive stage), learners benefit from seeing the whole task correctly performed. Dependence on visual inputs is high. Developing a reference of correctness (knowledge of results) is critical to ensure early skill acquisition (cognitive mapping).

126
Q
  1. A patient presents with pain and paresthesia over the first two metatarsal heads of her right foot. Pain is worse after prolonged periods of weight bearing. She typically wears shoes with 3-inch heels and pointed toes. What is the BEST intervention for this impairment?
    A. Pad placed proximal to the metatarsal heads
    B. Pad placed distal to the metatarsal heads
    C. Scaphoid pad to support the medial longitudinal arch
    D. Thomas heel to support the medial longitudinal arch
A

Correct Answer: A
Compression of the digital nerves in the forefoot results in sensory symptoms of pain and paresthesia (metatarsalgia). It is typically the result of excessively tight shoes. The best intervention is to wear larger shoes, with a metatarsal pad placed proximal to the metatarsal heads to elevate the transverse (anterior) arch and separate the metatarsals. Custom orthotics can also be molded to decrease load. Wearing of high heels should be discouraged. Stretching of plantarflexors may also be helpful.

127
Q
  1. A patient develop right throbbing should pain after painting the kitchen. Passive and active glenohumeral motions increase pain. What is the suspected diagnosis and BEST initial intervention?
    A. Stretching of the pectoralis minor muscle after acromioclavicular joint inflammation
    B. Manual therapy techniques and biophysical agents to reduce pain as the result of subdeltoid bursitis
    C. Rotator cuff strengthening exercises to allow ADL function after biceps tendinitis
    D. Correction of muscle imbalances to allow healing of right shoulder supraspinatus tendinitis
A

Correct Answer: B
Because pain occurs with both AROM and PROM, bursitis is the most likely cause of dysfunction. Initial interventions should focus on reducing pain and inflammation, Biophysical agents and manual therapy are the best choices.

128
Q
  1. A patient has a 5-year history of acquired immunodeficiency syndrome (AIDS). The caseworker reports a gradual increase in difficulty with walking. The patient rarely goes out anymore. A referral to PT is initiated. Examination findings reveal typical neuromuscular changes associated with AIDS. These deficits would likely include:
    A. Widespread sensory loss resulting in sensory ataxia
    B. Motor ataxia and paresis with pronounced gait disturbances
    C. Progressive rigidity and akinesia with severe balance disturbances
    D. Paraplegia or tetraplegia
A

Correct Answer: B
Alterations in memory, confusion, and disorientation are characteristics of AIDS dementia complex, a common central nervous system (CNS) manifestation of human immunodeficiency virus (HIV) infection. Motor deficits may include ataxia, paresis with gait disturbances, and loss of fine motor coordination, Patients may also develop peripheral neuropathy with distal pain and sensory loss.

129
Q
  1. A patient is referred by an orthopedist with a diagnosis of impingement syndrome of the shoulder, The initial PT examination reveals signs and symptoms that are not consistent with this diagnosis and are more consistent with thoracic spine pain and dysfunction. The therapist treats the patient consistent with PT findings without communicating with the referring physician. Months later, the therapist is sued by the patient’s estate. The patient died of undiagnosed metastatic lung cancer. The therapist is:
    A. Not responsible for diagnosing metastatic cancer, therefore cannot be held responsible for the patient’s death
    B. Responsible for communicating PT examination results to the referring physician
    C. Not responsible for the incorrect diagnosis because treatment was appropriate for the PT findings
    D. Responsible for making the diagnosis of possible cancer consistent with the PT examination of the patient
A

Correct Answer: B
When a referral relationship exists with another health care professional, it is the PT’s responsibility to communicate with the referring practitioner regarding the physical therapy examination, treatment plan, and management of the referred patient. This is particularly crucial when the findings are inconsistent with the referrer’s diagnosis.

130
Q

A PT is treating a 2-year-old child with Down syndrome who frequently uses a W sitting position. The main reason to discourage W sitting is that it may cause:
A. Hip subluxation and lateral knee stress
B. Femoral antetorsion and medial knee stress
C. Abnormally low tone because of reflex activity
D. Developmental delay of normal sitting

A

Correct Answer: B
W sitting is a stable and functional position, but may cause later orthopedic problems of femoral antetorsion and knee stress. Low muscle tone and difficulty with achieving sitting are reasons why these children choose to sit in a stable W sit position. Children with Down syndrome typically exhibit low tone and hyperextensibility.

131
Q
  1. A patient is being examined for impairments after stroke, When tested for two-point discrimination on the right hand, the patient is unable to tell whether the therapist is touching with one or two points. The therapist determines that there is impaired function in the:
    A. Spinotectal tract and somatosensory cortex
    B. Lateral spinothalamic tract and somatosensory cortex
    C. Dorsal column/lemniscal pathways and somatosensory cortex
    D. Anterior spinothalamic tract and thalamus
A

Correct Answer: C
Discriminative touch, proprioceptive sensibility, and vibration sense are carried in the posterior white columns (fasciculus cuneatus for the upper extremity and fasciculus gracilis for the lower extremity). The long ascending tracts cross the medulla (sensory decussation) and form the medial lemniscus, which then travels to the thalamus (ventral posterolateral nucleus) and finally to the cortex (postcentral gyrus). Loss of two-point discrimination could result from an insult affecting any of these component parts. Parietal lobe or internal capsule lesions are the most common sites.

132
Q
  1. An elderly patient has a history of two myocardial infarctions (MIs) and one episode of recent congestive heart failure (CHF). The patient also has claudication pain in the right calf during an exercise tolerance test. An initial exercise prescription that BEST deals with these problems is walking:
    A. Daily, using interval training for 10- to 15-minute periods.
    B. Five times a week using continuous training for 60 minutes.
    C. Three times a week using continuous training for 40-minute sessions.
    D. Three times a week using interval training for 30-minute periods.
A

Correct Answer: A
An appropriate initial exercise prescription for a patient with a history of CHF and claudication pain in the right calf should include low-intensity exercise (walking), low to moderate duration
(10-15 min), and higher frequencies (daily). The exercise session should carefully balance activity with rest (interval or discontinuous training).

133
Q
  1. A patient presents with signs and symptoms consistent with sacroiliac dysfunction. The cluster of special tests/ findings that has the highest diagnostic accuracy for sacroiliac dysfunction would be:
    A. Thigh thrust test, Gillet’s test, stork test, and Patrick’s test.
    B. Anterior superior iliac spine asymmetry, posterior iliac spine asymmetry, pubic symphysis pain with palpation, and sacral inferior lateral angle asymmetry.
    C. Fortin finger test, torsion test, supine-to-sit test, and Gaenslen’s test.
    D. SI gapping, sacroiliac compression, thigh thrust test (P4), sacral thrust, and Gaenslen’s test.
A

Correct Answer: D
It was found that if these 5 tests were clustered together as a group and at least 3/5 had a positive finding, they were found to have a high diagnostic accuracy.

134
Q
  1. A patient demonstrates postpartum sacral pain. The patient complains that pain is increased with prolonged walking, ascending or descending stairs, and rising from sit-to-stand. The intervention that is MOST beneficial for this problem is:
    A. Manual therapy techniques of the SI joint to provide relief of symptoms, and therapeutic exercise to restore normal function of the pelvic girdle.
    B. Performing mobilization followed by cryotherapy to restore normal motion to the SI
    joint.
    C. Cryotherapy and TENS to promote normal healing.
    D. Increasing non-weight bearing with ambulation training and stabilization using a lumbosacral orthosis.
A

Correct Answer: A
Ligamentous laxity and pain during pregnancy secondary to hormonal influences (relaxin) most commonly affects the SI joint, This ligamentous laxity continues to occur for up to 3 months after pregnancy and leaves the pelvic area vulnerable to injury. SI pain is aggravated by prolonged weight bearing and stairs. Manual therapy techniques are effective for reducing pain and therapeutic exercise is beneficial to restore normal muscle function.

135
Q
  1. A patient complains of right shoulder pain since falling onto the right shoulder 3 weeks ago. There was no dislocation and x-rays were negative. AROM is 35° of flexion and abduction with scapular elevation noted. Passive ROM is nearly full with mild pain and muscle guarding at the end of range. Resisted abduction is weak with pain noted in the anterior and lateral deltoid region. There is no atrophy. Based on the above findings, the physical therapist should MOST LIKELY suspect:
    A. Rotator cuff tear
    B. Axillary nerve palsy
    C. Supraspinatus tendinitis
    D. Adhesive capsulitis
A

Correct Answer: A
A rotator cuff tear would be provoked by testing because it is a contractile lesion. The patient would not be able to raise the arm over the head because of lack of force transmission secondary to the tear. Also, the mechanism of injury (trauma) could cause a tear.

136
Q
  1. A patient presents with neck pain, which is a result of a motor vehicle accident (hit from behind while the car was at rest). To determine the function of the deep cervical flexors, the physical therapist decides to perform a muscle function test utilizing the cranio-cervical flexion test. Findings of a normal test would be:
    A. When palpating the anterior cervical musculature during the active chin tuck, the sternocleidomastoid muscle activates prior to the longs colli muscle.
    B. During active chin tuck, the patient is able to hold the head 1 inch above the table for 30 seconds.
    C. During active chin tuck, the patient is able to maintain the normal cervical lordosis for 10 seconds
    D. During active chin tuck, the pressure in the stabilizer cuff increases to 22 and the patient can hold this position for 10 seconds.
A

Correct Answer: D
During an active chin tuck, the pressure in the stabilizer cuff increases to 22 and the patient can hold this position for 10 seconds.

137
Q

An elderly and frail older adult has low vision. The patient recently returned home from a
2-week hospitalization for stabilization of diabetes. The PT’s goal is to mobilize the patient and increase ambulation level and safety. The BEST intervention strategy for this patient is to:
A. Practice walking in areas of high illumination and low clutter.
B. Color-code stairs with pastel shades of blue and green to highlight steps.
C. Practice walking by having the patient look down at all times.
D. Keep window shades wide open to let in as much light as possible.

A

Correct Answer: A
Effective intervention strategies for the elderly patient with diabetes and low vision include ensuring adequate lighting. Vision and safety decrease dramatically in low lighting. Reducing clutter in the home is also an important strategy to improve safety during ambulation.

138
Q
  1. The patient has a history of angina percotis and limited physical activity. As the patient participates in the second exercise class, the PT suspects that angina is unstable and may be indicative of a preinfarction state. The therapist determines this by the presence of:
    A. Prolonged cessation of pain following the administration of nitroglycerin for angina
    B. Angina that responds to rest and interval training but not to continuous training.
    C. Arrhythmias of increasing frequency, especially atrial arrhythmias.
    D. Angina of increasing intensity that is unresponsive to the nitroglycerin or rest.
A

Correct Answer: D
Preinfarction or unstable angina pectoris is unrelieved by rest or nitroglycerin (measures that typically reduce most angina). The pain is described as increasing in intensity. Unstable angina is an absolute contraindication to exercise.

139
Q

A patient with a left CVA exhibits right hemiparesis and strong and dominant hemiplegic synergies in the lower extremity. Which activity would be BEST to break up these synergies?
A. Foot tapping in a sitting position
B. Supine, PNF D2F with knee flexing and D2E with knee extending.
C. Supine-lying, hip extension with adduction
D. Bridging, pelvic elevation.

A

Correct Answer: D
The typical lower extremity synergies are effectively broken up using bridging (combines hip extension from the extensor synergy with knee flexion from the flexion synergy).

140
Q

An elderly and frail resident of a nursing home has developed a stage III pressure ulcer.
The wound is open with necrosis of the subcutaneous tissue down to the fascia. This elderly patient when compared with a younger patient with the same type of ulcer can be expected to demonstrate:
A. Decreased vascular and immune responses resulting in impaired healing.
B. Increased vascular responses with significant erythema
C. Increased elasticity and eccrine sweating
D. Increased scarring with healing

A

Correct Answer: A
Age-associate changes in the integumentary system include decreased vascular and immune responses that result in impaired healing. Rate of healing is considerably slower,

141
Q

The therapist is examining a patient recovering from stroke for the expected pattern of spastic hypertonia in the more involved upper extremity. The therapist expects to find abnormally increased resistance to PROM in:
A. Shoulder adductors, forearm pronators, and elbow extensors.
B. Shoulder retractors and abductors and flexors of the elbow, wrist, and hand.
C. Shoulder flexors and flexors of the elbow and hand.
D. Shoulder adductors; forearm pronators; and flexors of the elbow, wrist, and hand.

A

Correct Answer: D
The typical pattern of spasticity in the UE of the patient recovering from stroke is shoulder adductors; forearm pronators; and flexors of the elbow, wrist, and hand (antigravity muscles).

142
Q

An elderly patient has been hospitalized for 3 weeks after a surgical resection of carcinoma of the colon, The patient is very weak and is currently receiving physical therapy to improve functional ambulation. During the initial sessions, the patient complains of pain in the left shoulder that is aggravated by weight bearing when using the walker. The therapist decides to:
A. Ambulate the patient in the parallel bars considering age and diagnosis.
B. Apply pulsed US to decrease pain.
C. Notify the physician immediately
D. Apply heat in the form of a hot pack before ambulation

A

Correct Answer: C
The risk of metastatic disease is present; the therapist should notify the physician immediately.

143
Q

Apatient sprained the left ankle 4 days ago. The patient complains of pain (4/10), and there is moderate swelling that is getting worse, At this time, which intervention would be
BEST to use?
A. Cold/intermittent compression combination with the limb elevated
B. Cold whirlpool followed by massage
C. Contrast baths followed by limb elevation
D. Intermittent compression followed by elevation

A

Correct Answer: A
The combination of RICE (rest, ice, compression, elevation) is best. Cold to decrease pain along with intermittent compression and elevation to facilitate fluid drainage provides the best intervention. Rest is required.

144
Q

A patient diagnosed with left lateral epicondylitis has no resolution of symptoms after 2 weeks of treatment. The PT reexamines the patient and finds the left biceps reflex is 1+. The therapist should NEXT perform a complete examination of the:
A. Cervicothoracic region
B. Upper cervical region
C. Cervicocranial region
D. Mid cervical region

A

Correct Answer: D
The patient has symptoms (diminished reflex of a possible left C5 nerve root compression in the mid cervical spine. Any reflex change suggests nerve root irritation or compression. Lateral epicondylitis frequently involves the extensor carpi radialis brevis, which is innervated by spinal nerves emanating from the mid cervical region.

145
Q

Which measure of aerobic capacity is supported as a reliable and valid functional capacity measure with reported minimal detectable change scores in patients with heart failure?
A. 6-Minute Walk Test.
B. Bruce protocol treadmill test.
C. Canadian Aerobic Fitness Step test.
D. Lower extremity ergometer test.

A

Correct Answer: A
Only the 6-Minute Walk Test (6MW T) has minimal detectable change (MDC) scores associate with it.

146
Q
  1. A patient underwent a right total hip replacement (THR) 4 months ago. The patient is now referred to physical therapy for gait evaluation. The patient demonstrates shortened stride length on the right. This patient MOST LIKELY has:
    A. Weakened quadriceps
    B. Contracted hip flexors
    C. Weakened hip flexors
    D. Contracted hamstrings
A

Correct Answer: B
Patients are less active after surgery and spend less time in standing and more time in sitting.
The iliopsoas muscles become shortened with increased time in sitting. The contracted iliopsoas limits the patient’s ability to extend the hip, which effectively shortens the stride length on the affected side.

147
Q
  1. A child with spastic cerebral palsy is having difficulty releasing food from the hand to the mouth. Once the child has brought the foot to the mouth, it would be helpful for the caregiver to:
    A. Slowly stroke the finger extensors in a proximal-to-distal direction
    B. Apply a quick stretch to the finger flexors
    C. Slowly stroke the finger flexors in a distal-to-proximal direction
    D. Passively extend the fingers
A

Correct Answer: A
Slowly stroking the finger extensors will help to facilitate opening of the hand and allow the child to release the food into the mouth.

148
Q

During the examination of the cervical spine of a client for C5 radiculopathy, small groupings of nevi are noted near the superior angle of the left scapula. The NEXT action the therapist should take is:
A. Perform a vertebral artery examination
B. Photograph the area in order to provide baseline documentation for the patient’s record.
C. Ask the patient about any history of moles and examine them closely.
D. Contact the physician immediately

A

Correct Answer: C
Nevi (moles) should be examined for asymmetry, border irregularities, color, and diameter (>6 mm), it is not uncommon to have a group of moles and they are usually benign, but if there is a transformation of a nevus (plural, nevi), then the primary care physician should be contacted. In this situation, the therapist needs to establish a baseline (history and physical examination) of the moles, and then determine whether there is an indication to contact the physician.

149
Q
  1. In reference to the figure, when lifting a constant load using either a stoop lift or a squat dift posture, the MOST significant contributing factor for increasing lumbar spine compression forces in addition to the weight of the load is:
    A. Performing the lift with the lumbar spine in a kyphotic posture
    B. Performing the lift with the lumbar spine in a neutral position rather than in a lordotic
    posture
    C. The height of the load from the ground
    D. The distance of the load from the base of the spine
A

Correct Answer: D
Manual lifting biomechanical models have demonstrated high lumbar spine moments, especially when the load is not held close to the body.

150
Q

Nearly 2 months ago, a patient noticed left shoulder pain after walking the dog. This pain has progressively worsened. The patient now is unable to move the left upper extremity overhead while performing ADLs. An orthopedic surgeon diagnosed the problem as adhesive capsulitis. The MOST effective direction for glenohumeral mobilization for this (patient would be:
A. Posteroinferior translatory glides.
B. Anteroinferior translatory glides.
C. Anterosuperior translatory glides
D. Posterosuperior translatory glides

A

Correct Answer: A
The diagnosis is left shoulder adhesive capsulitis. Inferior glides will improve the abduction and flexion (overhead motion). Posterior glides have been shown to be the most effective glide to increase glenohumeral external rotation. This is an exception to the concave-convex rule.

151
Q
  1. An elderly patient is referred to physical therapy for an examination of functional mobility skills and safety in the home environment. The family reports that the patient is demonstrating increasing forgetfulness and some memory deficits, From the examination, the therapist would expect to find:
    A. Impairments in short-term memory
    B. Periods of fluctuating confusion
    C. Periods of agitation and wandering, especially in the late afternoon
    D. Significant impairments in long-term memory
A

Correct Answer: A
Elderly patients with memory impairments typically demonstrate intact immediate recall (e.g., can repeat words); impairments are often noted in memory for recent events (e.g., Why did I come into this room? Who came to see me yesterday?). Long-term memory is usually intact.

152
Q

A therapist is examining a patient with vestibular dysfunction. The patient is asked to assume a long sitting position with the head turned to the left side. The therapist then quickly moves the patient backward so that the head is extended over the end of the table approximately 30° below horizontal, This maneuver causes severe dizziness and vertigo. A repeat test with the head turned to the right produces no symptoms. What is the BEST way to document these results?
A. Positive left Hallpike-Dix test
B. Positive sharpened Romberg’s test
C. Positive right positional test
D. Positive positional test

A

Correct Answer: A
The test described is the Hallpike-Dix. It is a left positive test because, with the head turned to the left, the change in position produces the patient’s symptoms.

153
Q
  1. A therapist is instructing a patient with a stroke in gait training. The therapist determines that learning is going well because the patient’s errors are decreasing and overall endurance is improving. The BEST strategy to promote continued motor learning at this point in the patient’s rehabilitation is to:
    A. Have the patient practice walking in varying environments.
    B. Intervene early whenever errors appear before bad habits become firmly entrenched
    C. Provide continuous feedback after every walking trial
    D. Have the patient continue to practice in the parallel bars until all errors are extinguished
A

Correct Answer: A
This patient demonstrates the associative stage of motor learning (errors are decreasing and movements are becoming organized). It is appropriate to gradually progress this patient toward ambulating in a more open (varied) environment.

154
Q

A young child with newly diagnosed cystic fibrosis is being seen by a PT in the home.
Which intervention should be considered for this patient?
A. Teach the child use of the acapella device in postural drainage positions to be performed once or twice a day
B. Teach the child autogenic drainage for secretion removal to be performed once or twice daily
C. Teach the parents secretion removal techniques to all segments of all lobes of both lungs once or twice a dav
D. Teach the child active cycle of breathing technique (ACBT) to be done once or twice a day to clear retained secretions

A

Correct Answer: C
For a young child, proper at-home interventions include secretion removal techniques including manual techniques performed by an adult once or twice a day.

155
Q
  1. A middle-aged woman is referred to a women’s clinic with problems of stress incontinence. She reports loss of control that began with coughing or laughing, but now reports problems even when she exercises (aerobics 3 times/wk). the BEST intervention for this patient is:
    A. Kegel’s exercises several times a day
    B. Behavioral modification techniques to reward proper voiding on schedule
    C. Biofeedback 1 hour/wk to achieve appropriate sphincter control
    D. Functional electrical stimulation 3 times/wk
A

Correct Answer: A
Symptoms of stress incontinence can be successfully managed through a variety of techniques. Pelvic floor exercises (Kegel’s exercises) are the mainstay of treatment and must be performed daily, several times a day, in order to be effective.

156
Q

An elderly patient suffered a cerebral thrombosis 4 days ago and presents with the following symptoms: decreased pain and temperature sensation of the ipsilateral face, nystagmus, vertigo, nausea, dysphagia, ipsilateral Horner’s syndrome, and contralateral loss of pain and temperature sensation of the body, The MOST LIKELY site of the thrombosis is the:
A. Anterior cerebral artery
B. Posterior inferior cerebellar artery
C. Internal carotid artery
D. Posterior cerebral artery

A

Correct Answer: B
This patient presents with lateral medullary (wallenberg’s) syndrome, which can result from occlusion of the PICA, which is usually a branch of the vertebral artery. It involves the descending tract and nucleus of CN V, the vestibular nucleus and its connections, CN IX and CN X nuclei or nerve fibers, cuneate and gracile nuclei and spinothalamic tract.

157
Q
  1. A patient with a 2-inch stage II decubitus ulcer over the left lateral malleolus is referred for physical therapy. The therapist notes a greenish, pungent exudate at the wound site. The therapist decides to use electrical stimulation. The BEST choice of polarity and electrode (placement is?
    A. Cathode placed in the wound
    B. Cathode placed proximal to wound
    C. Anode placed in the wound
    D. Anode placed proximal to wound
A

Correct Answer: A
It is purported that the bactericidal effect produced by negative current is a result of substrate depletion or alteration of the internal processes of the microorganisms. Neutrophils are also attracted to the wound area by chemotaxis to purge the bacteria. The cathode should be placed directly in contact with the wound to cover as much treatment are as possible.

158
Q

A patient arrives for outpatient cardiac rehab 10 weeks after a coronary artery bypass graft. The postoperative course was complicated by atrial fibrillation, which has been controlled with medications prescribed by a cardiologist. The patient’s resting vital signs are
HR=90 in atrial fibrillation, BP= 116/74, RR= 14, and SpO2 = 99% on room air. Is a
symptom-limited exercise test appropriate for this patient at this time?
A. Yes, the patient is managed by a cardiologist and has no symptoms now
B. Yes, the heart rate is well controlled and the cardiologist is aware of the arrhythmia
C. No, the patient is tachycardic at rest and has an arrhythmia
D. No, a patient in atrial fibrillation should not complete an exercise test

A

Correct Answer: B
It is appropriate to perform an exercise test on a patient who is no medication to control his/her rate with atrial fibrillation (AF). A normal heart rate is between 60-100 beats per minute, which is where this patient’s heart rate falls.

159
Q

A patient presents with a complete T10 paraplegia. An extensive neurological workup has failed to reveal specific cause for the paraplegia. The physician has determined a diagnosis of conversion disorder. During physical therapy, it would be BEST to:
A. Initiate ROM and strength training after the patient receives psychological counseling
B. Initiate functional training consistent with the level of injury
C. Use functional electrical stimulation as a means of demonstrating to the patient that the muscles are functional.
D. Discuss possible underlying cause for the paralysis with the patient in an empathetic
manner

A

Correct Answer: B
A conversion disorder (hysterical paralysis) represents a real loss of function for the patient.
The therapist should treat this patient the same as any patient with spinal cord injury with similar functional deficits. Early intervention is crucial.

160
Q

The director of a physical therapy department wants to fill a vacant PT position in the spinal cord injury program. Two resumes have been received. One candidate, a former employee, is a well-qualified and experienced 52-year-old female with a history of back pain which could impact her ability to do some heavy lifting at times. The other candidate is a newly licensed, very enthusiastic, 25-year-old male therapist for whom heavy lifting should not be a problem. In this case, what is the BEST hiring decision?
A. As long as age and back pain history were NOT discussed during the interview process with the female candidate, the male candidate would best meet the case load demands
B. Hire the male candidate but ensure that age and back pain history were discussed with the female candidate as the rationale for hiring someone else
C. Not have the female candidate partake in the interview process as issues with age and back pain would be justifiable grounds to rule her out based on the case load
D. Hire the more qualified female and provide aide assistance or lift equipment when heavy lifting is required.

A

Correct Answer: D
The Age discrimination and employment act of 1967 prohibits employers from discriminating against persons 40-70 years of age in any area of employment. The 1973 Rehabilitation Act prohibits employment discrimination based on disability and requires reasonable accommodation in the workplace by removing barriers unless there would be “undue hardship” for the employer. Title VII of the Civil Rights Act of 1964 might also come into play since one provision prohibits discrimination based on gender. The female candidate is clearly the more qualified and would be the best hire in this case.

161
Q

Which is an abnormal finding during an examination of a newborn infant?
A. Continuous tremulousness
B. Response decrement to repetitive stimuli
C. Dramatic skin color changes with change of state
D. Symmetry in ROM

A

Correct Answer: A
Continuous tremulousness is an abnormal finding, but occasional tremulousness is not.

162
Q

A patient with long-standing traumatic brain injury (TBI) comes into an out patient clinic using a standing wheelchair. The patient demonstrates sacral sitting with a rounded, kyphotic upper back. The therapist suspects the cause of this posture is:
A. Decreased floor to seat height
B. Uneven weight distribution on the thighs and ischial seat
C. Excessive leg length from seat to the foot plate
D. Excessive seat width

A

Correct Answer: C
Excessive leg length on a wheelchair can result in sliding forward in the wheelchair to reach the foot plate. This results in a posterior tilt of the pelvis and sacral sitting.

163
Q
  1. A patient presents with a large sacral decubitus ulcer that is purulent and draining. The therapist needs to take a representative sample of the infected material in order to obtain a aboratory culture. The method to culture this wound is to obtain samples from the:
    A. Exudate in the wound and the surrounding tissues
    B. Dressing and exudate in the wound
    C. Exudate in the wound
    D. Dressing, exudate, and surrounding bed linen
A

Correct Answer: C
The specimens must be collected from the wound site with a minimum of contamination by material from adiacent tissues. The exudate provides the best culture.

164
Q

A phase 2 outpatient cardiac rehabilitation program uses circuit training with different exercise stations for the 50-minute program. One station uses arm ergometry. For arm exercise as compared with leg exercise, at a given workload, the PT can expect:
A. Higher systolic and diastolic BP
B. Higher HR and systolic/diastolic BP
C. Higher HR and lower systolic BP
D. Reduced exercise capacity owing to higher stroke volumes

A

Correct Answer: B
Arm ergometry uses a small muscle mass than leg ergometry, with resulting lower maximal oxygen uptake. In upper extremity exercise, both HR and BP will be higher than for the same level fo work in the lower extremities.

165
Q

A therapist is working in a major medical center and is new to the acute care setting. An orientation session for new employees concerns infection control. The therapist recognized that the most common infection transmitted to health care workers is:
A. Tuberculosis
B. Human immunodeficiency virus (HIV)
C. Hepatitis A
D. Hepatitis B

A

Correct Answer: D
Health care workers are most likely to contract hepatitis B (estimated incidence 300,000 new acute cases in the United States each year). Transmission is through exposure to blood and blood products and infected body fluids.

166
Q

Before liver transplantation, a patient had a body mass index (BMI of 17 and generalized muscle atrophy and completed the 6-minute walk with 65% of age-predicted distance, Surgery was 10 days ago, and the patient is able to complete bed mobility with an overhead trapeze, walk independently for short distances with a rolling walker, and complete deep breathing and lower extremity AROM exercises for two sets of 10 repetitions. The patient is being discharged home with family assistance today. Home care physical therapy is scheduled to begin in 1 week. The BEST choices for discharge home exercise program is:
A. Stationary cycling and lower extremity resistance exercises using a 5-1b weight cuff
B. Independent bed mobility exercises, elastic resistance extremity exercise, and partial sit-ups
C. Independent ambulation, elastic resistance lower extremity exercise, and active abdominal strengthening
D. Breathing exercises, ambulation with walker, and AROM lower extremity exercises.

A

Correct Choices: D
The in-hospital program should be continued until the home care therapist can make his/her own assessment and plan of care, The postoperative goals of improved ventilation, assisted mobility, and AROM are appropriate for the 3-week postoperative time period, considering the debilitation before surgery and the abdominal surgery.

167
Q

Based on the pictured CT scan, this patient is LIKELY to manifest:
A. Ataxia
B. Nonfluent aphasia
C. Fluent aphasia
D. Left-sided unilateral neglect

A

Correct Answer: A
The arrow is pointing to a hemorrhage in the cerebellum. Damage to this area results in difficulty with movement, postural control, eye-movement disorders, and muscle tone, Ataxia is a common finding.

168
Q

A child with a developmental disability continues to be a no show for PT scheduled appointments. When the child does arrive to PT the family is usually late, It does not seem as though the home exercise program is being done and the child is regressing quickly. After the physical therapist discusses this with the parents they get angry and stop bringing the child for treatment. The BEST course of action in this situation would be to:
A. Report the family for neglect of the child
B. Contact your supervisor with information that you are discharging this child
C. Do nothing because it is the family’s choice whether to continue or not
D. Report the family for abuse of the child

A

Correct Answer: A
Withholding necessary medical care from a child fits the definition of neglect. This is often a tough call, however, this child is regressing in ability and it seems that it is based on the parents’ behavior.

169
Q

The most efficient intervention to regain biceps brachi strength if the muscle is chronically inflamed and has painful arc of motion is:
A. Active concentric contractions through partial ROM
B. Active eccentric contractions in the pain-free range
C. Isokinetic exercises through the full ROM
D. Isometric exercises at the end range of movement only

A

Correct Answer: B
For a muscle that is chronically inflamed, focus should be placed on eccentric contraction, because there is less effort and stress placed on the contractile units than with concentric contractions at the same level of work. The exercise should be performed in the pain-free portion of the range.

170
Q

During an ultrasound (US) treatment, the patient flinches and states that a strong ache was felt in the treatment area. To address this patient’s concern, it would be BEST to:
A. Decrease the US frequency
B. Add more transmission medium
C. Decrease the US intensity
D. Increase the size of the treatment area

A

Correct Answer: C
Acoustical energy is reflected from the bone into the bone-tissue interface, resulting in rapid tissue temperature elevation and stimulation of the highly sensitive periosteum of the bone. A reduction in intensity is indicated if a strong ache is felt.

171
Q

A patient is referred for postoperative rehabilitation following a type II SLAP repair performed 1 week ago. The physical therapist should:
A. Focus on biceps brachi stretching and strengthening
B. Defer intervention during the maximum protection phase
C. Perform careful ROM of the shoulder internal rotators
D. Perform careful ROM of the shoulder external rotators

A

Correct Answer: C
Internal rotation ROM does not create the peel back mechanism that increases stress to the repair, Given the nature of this repair, an understanding of the postoperative precautions is paramount to a successful surgical outcome. Early rehabilitation within the postoperative precautions correlates to a quicker overall recovery and improved outcomes.

172
Q

A patient is recovering from deep-partial-thickness burns over the posterior thigh and calf that are now healed. The therapist’s examination reveals local tenderness with swelling and pain on movement in the hip area. While palpating the tissues, the therapist detects a mass.
The therapist’s BEST course of action is to:
A. Use petrissage to work on this area of focal tenderness
B. Report these findings promptly to the physician
C. Continue with ROM exercise but proceed gently
D. Use RICE to quiet down the inflammatory

A

Correct Answer: B
These signs and symptoms are characteristic of heterotopic ossification (HO), an abnormal bone growth typically around a joint. While the etiology is unknown, its presence can lead to serious ROM limitations. These findings should be reported promptly to the physician.

173
Q

A patient presents with complaint of neck pain on the right. During the AROM examination, the physical therapist observes the following osteokinematic neck motions full side-bending left, full rotation to the left, full forward flexion, limited and painful extension, limited and painful right side-bending, and limited and painful right rotation.
Based on this pattern, what is the arthrokinematic restriction?
A. Restriction with upglide of a facet on the left
B. Restriction with upglide of a facet on the right
C. Restriction with downglide of a facet on the left
D. Restriction with downglide of a facet on the right

A

Correct Answer: D
If a facet on the right was restricted with downgliding (arthrokinematic restriction), then the osteokinematic motions that would be limited would be rotation and side-bending to the right with limited extension. The fact that there is pain on the right supports that the restriction is on the right.

174
Q

ECG changes that may occur with exercise in an individual with coronary artery disease
(CAD) and prior myocardial infarction (MI) include:
A. Tachycardia at a relatively low intensity of exercise with ST segment depression
B. Bradycardia with ST segment elevation
C. Significant arrhythmias early on in exercise with a shortened QRS
D. Bradycardia with ST segment depression > 3 mm below baseline

A

Correct Answer: A
The typical exercise ECG changes in the patient with CAD include tachycardia at low levels of exercise intensity. The ST segment becomes depressed (>1 mm is significant). In addition, complex ventricular arrhythmias (multifocal or runs of PVCs) may appear, and are associated with significant CAD and/or a poor prognosis.

175
Q

A patient with long-term postural changes exhibits an excessive forward head, and complains of pain and dizziness when looking upward. The MOST effective physical therapy intervention is:
A. Manual therapy techniques to provide pain relief and postural reeducation
B. Postural reeducation to reduce compression of the cervical sympathetic ganglia
C. Strengthening exercises to the posterior cervical musculature
D. Anterior cervical muscle stretching and postural reeducation to relieve vertebral artery compression

A

Correct Answer: A
Long-term postural changes with forward head posture include shortening of the posterior muscles, potential joint restrictions with possible vertebral artery compromise at the occiput.
Restoration of normal movement throughout the cervical region and postural reeducation is the best choice for this condition.

176
Q
  1. A patient with idiopathic dilated cardiomyopathy is on the cardiac unit with telemetry ECG monitoring after a recent admission for decompensated heart failure, Figure 1 depicts this patient’s resting telemetric ECG recording, and Figure 2 depicts the patient’s exercise (ambulating 2.5 mph on a flat surface in the hallway) telemetric ECG recording. What is an accurate interpretation of the change between the two recordings?
    A. Ventricular tachycardia, indicating abnormal response to increasing work demands and moderate risk.
    B. Tachycardia, indicating normal response to increasing work demands and normal risk.
    C. Bradycardia, indicating abnormal response to increasing work demands and high risk
    D. Preventricular contractions, indicating abnormal response to increasing work demands and high risk.
A

Correct Answer: D
New onset of preventricular contractions (wide complex, lack of a PorT wave) indicates an abnormal response to increasing work demands and indicates high risk for cardiac patients based on American College response to increasing work demands and indicates high risk for cardiac patients based on American College of Physicians and American Association of Cardiovascular and Pulmonary Rehabilitation risk stratification.

177
Q

During the course of the physical therapy treatment in the ICU, a radial artery line gets pulled (comes out of the artery), What is the first action the PT should take?
A. Push the code button in the patient’s room, because this is a cardiac emergency
B. Elevate the arm above heart level to stop the bleeding
C. Place a BP cuff on the involved extremity and inflate the cuff until the bleeding stops
D. Reinsert the arterial catheter into the radial artery and check the monitor for an accurate
tracing

A

Correct Answer: C
A radial arterial line is a catheter placed in the artery itself. If it becomes dislodged during treatment, the artery is now open to bleeding. This arterial bleeding needs to be stopped immediately, although it is not considered a cardiac emergency. Place a BP cuff above the site of bleeding and inflate the cuff to above systole to stop the bleeding or place enough manual pressure on the site to stop the bleeding. Then call for help.

178
Q

A therapist working in an outpatient clinic examines a patient referred for exercise conditioning. During the intial examination, the therapist finds unusual swelling and enlargement in the anterior neck with mild tenderness. The patient does not have any hoarseness or difficulty swallowing. What is the therapist’s BEST course of action?
A. Notify the referring physician
B. Document the findings in the medical record
C. Take girth measurements of the neck
D. Initiate the plan of care

A

Correct Answer: A
This patient is likely exhibiting hyperthyroidism (graves’ disease) and should be referred to the physician of record. Additional manifestations of hyperthyroidism include cardiopulmonary changes (increased HR and respiratory rate, palpitations, dysrhythmias, breathlessness), CNS changes (tremors, hyperkinesias, nervousness, increased DTRs), muscyloskeletal changes (weakness, fatigue, atrophy), and integumentary effects (heat intolerance).

179
Q

A patient with a methicillin-resistant staphylococcus aureus (MRSA) infection has been discharged from an isolation setting with an open wound of the buttocks. The patient is now returning to physical therapy as an outpatient. The therapist should adhere to which precaution?
A. An open wound must be contained within a dressing
B. Direct contact with the patient should be avoided
C. Gloves are needed only with dressing changes
D. Treatment can be performed in the therapy gym if contact surfaces are covered

A

Correct Answer: A
Staphylococcal organisms are spread by contact. Open wounds must be well contained with a dressing. Standard germicidal cleaning measures (hand washing) should be followed. The therapist should be gloved for any direct contact with the patient’s surfaces or articles in close proximity to the patient. All equipment should be cleaned with an approved germicidal agent before and after use.

180
Q
  1. After examining a patient who was referred to physical therapy for posterior thoracic pain, the therapist finds no musculoskeletal causes for the patient’s symptoms. What anatomical structure may refer pain to this thoracic region?
    A. Heart
    B. Appendix
    C. Gallbladder
    D. Ovary
A

Correct Answer: C
Dysfunction of the gallbladder often refers pain to the thorax.

181
Q

An elderly patient with a transfemoral amputation is being fitted with a temporary prosthesis containing a SACH (solid ankle cushion heel) prosthetic foot. Which of the following BEST characterizes the SACH foot?
A. Is an articulated foot with multiplanar motion
B. Allows full sagittal and frontal plane motion
C. Absorbs energy through a series of bumpers, permitting sagittal plane motion only.
D. Allows limited sagittal plane motion with a small amount of mediolateral motion

A

Correct Answer: D
The SACH foot is the most commonly prescribed type of prosthetic foot. It provides for sagittal plane motion (primarily plantarflexion) and very limited frontal plane motion (mediolateral motion).

182
Q

physical therapist examines an adult patient that recently suffered a stroke that involved the right internal capsule, In addition to hemiparesis of the contralateral extremities, the patient also exhibits a facial palsy. Which facial muscles would MOST likely be affected?
A. All muscles on the left side of the face
B. All muscles on the right side of the face
C. Only muscles on the lower half of the right side of the face
D. Only muscles on the lower half of the left side of the face

A

Correct Answer: D
A stroke that involves the internal capsule would result in a supranuclear palsy, which affects only the contralateral lower half of the face. The specific pathway that is affected is the corticobulbar tract, which contains upper motor neurons (UMN) that project from the motor cortex to the nucleus of the facial nerve (cranial nerve VII) in the brainstem. The muscles in the upper half of the face are spared because both the right and left cerebral cortex project to the lower motor neurons (LMNs) in the facial nucleus that the innervate muscles of the forehead. In contrast, the LMNs that innervate muscles of the lower half of the face receive input from the contralateral motor cortex only. Therefore, a stroke that affects the internal capsule (corticobulbar tract) prevents input from the motor cortex to the contralateral facial nucleus, causing paresis or paralysis of the muscles of the lower half of the face only.

183
Q

A patient in the ICU is referred to physical therapy and presents with significant shortness of breath. Notable on physical examination is a deviated trachea to the left. Which of the following processes would account for such a finding?
A. Right lung collapse
B. Left pleural effusion
C. Right hemothorax
D. Left pneumothorax

A

Correct Answer: C
A right hemothorax (blood was in the pleural space) takes up space in the right hemothorax, shifting the trachea to the left.

184
Q

A patient complains of pain (7/10) in the shoulder region secondary to acute subdeltoid bursitis. As part of the plan of care during the acute phase, the therapist elects to use conventional TENS. Which of the following BEST identifies the modulating properties of this biophysical device?
A. Stimulation of endorphins
B. Gate control mechanisims
C. Descending inhibition
D. Ascending inhibition

A

Correct Answer: B
The gate control mechanism is activated by the application of conventional (high-rate) TENS at the spinal cord level.

185
Q

A patient described a sudden onset of back pain while trying to lift a heavy barrel. The patient described this pain as constant, unremitting at an intensity of 10/10 over the past 3 days, and unresponsive to pain medications. The patient is unable to work but is able to drive to the clinic for treatment unaided. There is no history of other back-related symptoms in the past. Which of the following is the MOST likely causative factor?
A. Early degenerative osteoarthritis
B. Herniated lumbar disc
C. Neoplastic disease
D. Secondary gain

A

Correct Answer: D
A patient who is able to drive to the clinic for treatment and relates pain level of 10/10 is not providing consistent subjective date. Secondary gain in this case, and not working, is a likely factor.

186
Q

After myocardial infarction (MI), a patient was placed on medications that included a beta-adrenergic blocking agent. When monitoring this patient’s response to exercise, what changes in HR are expected?
A. Increase proportionally to changes in diastolic BP
B. Be low at rest and rise very little with exercise
C. Increase proportionally to changes in systolic BP
D. Be low at rest and rise linearly as a function of increasing workload

A

Correct Answer: B
Beta-adrenergic blocking agents (e.g., propranolol [Inderal]) are used to treat hypertension, prevent angina pectoris, and prevent certain arrhythmias, In individuals taking these drugs, HR is low at rest and rises very little with exercise (blunted response. These changes, therefore, invalidate the use of HR to monitor exercise responses. A more sensitive measure would be RPE.

187
Q

An individual presents with chronic TMJ dysfunction. There is limited lateral movement of the mandible to the right as a result of muscular tightness. Which of the following muscles should be the focus of inhibitory or soft tissue lengthening techniques?
A. Right temporalis muscle
B. Right geniohyoid muscle
C. Right medial pterygoid muscle
D. Right digastric muscle

A

Correct Answer: C
Lateral excursion of the mandible is produced by the contralateral medial and lateral ptergoids and ipsilateral temporalis muscles. Tightness of the ipsilateral pterygoids or contralateral temporalis muscles may need lengthening in order to allow full lateral mandibular excursion.
The right medial pterygoid would be one of the muscles targeted for interventions restoring sufficient length.

188
Q
  1. A patient has a 20-year history of diabetes. Notable on the examination are the following vascular insufficiency and diminished sensation of both feet with poor healing of a superficial skin lesion. It is important that the patient understand the precautions and guidelines on foot care for people with diabetes. Which recommendation is
    CONTRAINDICATED to include in patient care instructions?
    A. Inspect the skin daily for inflammation, swelling, redness, blister, or wounds
    B. Wash the feet daily and hydrate with moisturizing lotion
    C. Wear flexible shoes that allow adequate room and change shoes frequently
    D. Use daily hot soaks and moisturize the skin
A

Correct Answer: D
Daily hot soaks are contraindicated because of the increased risk of thermal injury. The patient with diabetes typically has loss of protective sensations.

189
Q

After completing an examination of a patient with shoulder pain, the PT concludes that the cause is subscapularis tendinitis. Which clinical finding is supportive of this conclusion?
A. Tenderness at the greater tubercle of the humerus
B. Painful resisted shoulder adduction
C. Pain provoked with active glenohumeral external rotation
D. Pain provoked with passive glenohumeral external rotation

A

Correct Answer: D
The subscapularis is an internal rotator of the humerus. It will be painful if passively stretched into external rotation and irritated when contracting or being resisted when the shoulder internally rotates. The muscle inserts onto the lesser tubercle of the humerus and plays no role in shoulder adduction.

190
Q

A soccer player sustained a grade II inversion ankle sprain 2 weeks ago, What is the BEST intervention to use in the early subacute phase of rehabilitation?
A. Mobilization at the talocrural and subtalar joints
B. Closed-chain strengthening and proprioceptive exercises
C. Plyometric-based exercise program
D. Functional soccer-related drills

A

Correct Answer: B
The most effective treatment for this athlete would involve closed-chain exercises and proprioceptive training, appropriate interventions for early subacute phase management.

191
Q

A patient is referred for outpatient care after a tendon transfer of the extensor carpi radialis longus. The muscle strength tests poor (2/5) in spite of previous intensive therapy.
The therapist elects to apply biofeedback to assist in progressively increasing active motor recruitment. What is the BEST choices for the initial EMG protocol?
A. High-detection sensitivity with recording electrode placed far apart.
B. Low-detection sensitivity with recording electrodes placed close together.
C. High-detection sensitivity with recording electrodes placed close together.
D. Low-detection sensitivity with recording electrodes placed far apart.

A

Correct Answer: C
Initially, high-detection sensitivity is needed to detect low-amplitude signals generated by a small number of motor units such as in a weak extensor carpi radialis longus.

192
Q
  1. A PT receives a home care referral from the nurse case manager. An elderly man has lost functional independence after the recent death of his wife. His past medical history includes stroke with minimal residual disability. Currently, he no longer goes out of his house and rarely even gets out of his chair anymore. During the initial session, the therapist determines that depression may be the cause of his increasing inactivity. What clinical signs and symptoms would lead the therapist to reach this determination?
    A. Low scores on the Geriatric Depression Scale
    B. Weight loss and social withdrawal
    C. Complains of increasing dizziness and palpitations
    D. Sleep apnea and weight gain
A

Correct Answer: B
Depression is associated with symptoms of withdrawal, fatigue, and weight loss.

193
Q

Exercise, electrical stimulation and biofeedback methodology are MOST OFTEN employed by physical therapists in the management of which type of incontinence?
A. Urge incontinence
B. Functional incontinence
C. Overflow incontinence
D. Stress incontinence

A

Correct Answer: D
Exercises (Kegel), electrical stimulation, and biofeedback are used to increase the strength and functionality of the levator ani and other pelvic muscles in cases where increased intra-abdominal pressure results or can result in a sudden release of urine. This is called stress incontinence. Urine leaks due to weakened pelvic floor muscles and tissues.

194
Q

A patient has been on bed rest for 4 days following complications after revascularization surgery involving a triple coronary artery bypass graft. During the first therapy session, the patient complains of tenderness and aching in the right calf. For what clinical signs should the therapist immediately examine?
A. Lowered body temperature
B. Bradycardia
C. Swelling in the calf or ankle
D. Homan’s sign

A

Correct Answer: C
Deep vein thrombophlebitis (DVT) is characterized by classic signs of inflammation (tenderness, aching, and swelling), typically in the calf. Rapid screening is possible with Doppler ultrasonography. Color flow venous duplex scanning is the primary diagnostic test for detection of DVT.

195
Q

A patient is referred for physical therapy with jaw pain and dysfunction. The patient has experienced three episodes of jaw locking in an open position in the past week. What is the MOST likely cause of the patient’s jaw locking?
A. Entrapment of the retrodiscal lamina
B. Lateral pterygoid muscle spasm
C. Impingement of the temporomandibular ligament
D. Disc displacement

A

Correct Answer: D
This patient is experiencing temporomandibular joint dysfunctin (TMJ). The jaw becomes locked in an open position when the disc is displaced. The muscles influence lateral deviation of the jaw with opening.

196
Q
  1. An elderly patient is referred to physical therapy after a fall and ORIF for a fracture of the right wrist. During the initial examination, the therapist observes that the patient’s skin and eyes have a yellowish hue. What is the therapist’s BEST course of action?
    A. Send a copy of the examination results to the referring surgeon, emphasizing the skin hue
    B. Treat the problem with whirlpool and massage and reevaluate skin color posttreatment
    C. Continue with the treatment; a yellowish hue is an expected finding 3-4 days post-ORIF
    D. Document the findings and consult with the surgeon immediately after treatment
A

Correct Answer: D
This patient is most likely experiencing jaundice as a result of liver dysfunction, The therapist’s best course of action is to document the findings and consult with the surgeon immediately, preferably by phone.

197
Q
  1. An older adult received a cemented total hip replacement (THR) 2 days ago. What is the therapist’s initial priority?
    A. AROM exercises and early ambulation using a walker, non-weight bearing
    B. PROM exercises and gait training using crutches, weight bearing to tolerance
    C. Proper technique for transferring to the toilet
    D. Patient education regarding positions and movements to avoid
A

Correct Answer: D
Education regarding positions and movements to avoid is the number one priority. Standard hip precautions stress avoiding excessive flexion, internal rotation, and adduction.

198
Q

A patient with recent trauma presents with restricted movement of the right hand. There is decreased motion at the third right MCP joint. To differentiate as to whether this is joint restriction or some other type of tightness (not joint), which examination procedure should be employed?
A. Finkelstein’s test
B. Bunnel-Littler test
C. Tight retinacular test
D. Froment’s sign

A

Correct Answer: B
The Bunnel-Littler test is specifically utilized to determine if there is a joint restriction present at the MCP joints.

199
Q

A patient with stage II primary lymphedema of the right lower extremity is referred for physical therapy. Examination reveals increased limb girth with skin folds/flaps evident. An important component of lymphedema management is manual lymphatic drainage. Which of the following describes a cardinal principle of manual lymphedema management?
A. Deep tissue friction massage for several minutes on fibrotic areas.
B. Decongesting the proximal portions of the limb first and working distally
C. Decongesting the trunk after the limb segments
D. Decongesting the distal portions of the limb first and working proximally

A

Correct Answer: B
Lymphedema is a swelling of the soft tissues that occurs with an accumulation of protein-rich fluid in the extracellular spaces. Causes of primary lymphedema include developmental abnormalities, heredity, surgery, or unknown etiology. Stage Il lymphedema is characterized by nompitting edema with connective scar tissue and clinical fibrosis. Lymphatic drainage is assisted by manual stroking (e.g., Vodder, Leduc’s Foldi’s, Casley-Smith pressure techniques).
All techniques use cardinal principles: proximal limb segments before distal, trunk segments before limb segments, and directing the flow of the lymphatics centrally toward the lymphatic ducts.

200
Q

A patient with a 10-year history of multiple sclerosis (MS) demonstrates 3+ extensor tone in both lower extremities. The therapists needs to order a wheelchair. What is the BEST recommendation for this patient?
A. Standard wheelchair with elevating legrests
B. Tilt-in-space wheelchair with a pelvic belt
C. Standard wheelchair with a 30° reclining back
D. Electric wheelchair with toe loops

A

Correct Answer: B
A patient with strong extensor tone needs controls over the hips (pelvic belt) to maintain the hips in flexion, The tilt-in-space design best assists in keeping the patient from coming out of the chair when extensor spasms are active.